February 2018 Pennsylvania Bar Examination: Essay Questions and Examiners' Analyses and Performance Test

You might also like

Download as pdf or txt
Download as pdf or txt
You are on page 1of 74

FEBRUARY 2018

PENNSYLVANIA BAR
EXAMINATION

Essay Questions and Examiners’ Analyses


and
Performance Test

Pennsylvania Board of Law Examiners


601 Commonwealth Avenue, Suite 3600
P.O. Box 62535
Harrisburg, PA 17106-2535
(717) 231-3350
www.pabarexam.org

©2018 Pennsylvania Board of Law Examiners


Table of Contents

Index ..................................................................................................................................................ii

Question No. 1: Facts and Interrogatories, Examiner's Analysis and Grading Guidelines ..............1

Question No. 2: Facts and Interrogatories, Examiner's Analysis and Grading Guidelines ..............9

Question No. 3: Facts and Interrogatories, Examiner's Analysis and Grading Guidelines ..............16

Question No. 4: Facts and Interrogatories, Examiner's Analysis and Grading Guidelines ..............23

Question No. 5: Facts and Interrogatories, Examiner's Analysis and Grading Guidelines ..............33

Question No. 6: Facts and Interrogatories, Examiner's Analysis and Grading Guidelines ..............42

Performance Test and Grading Guidelines ........................................................................................50

i
Index

Question No. 1

1. Decedent’s Estates: oral trust

2. Decedent’s Estates: ademption

3. Federal Income Tax: capital gain on sale of residence

4. Professional Responsibility: lawyer competency

Question No. 2

1. Conflict of Laws: domicile

2. Business Organizations: partnership liability

3. U.C.C. Article II: statue of frauds

4. Business Organizations: limitation in partnership agreement

Question No. 3

1. Criminal Law: kidnapping

2. Criminal Law: aggravated assault

3. Family Law: annulment

4. Evidence: spousal communication privilege

Question No. 4

1. Constitutional Law: Privileges and Immunities Clause

2. Employment Law: Older Worker Benefit Protection Act

3. Employment Law / Federal Civil Procedure: summary judgment, age discrimination

ii
Question No. 5

1. Property: statute of frauds

2. Contracts: substantial performance, breach of contract

3. Contracts: promissory estoppel

4. Property: delivery of deed

Question No. 6

1(a). Torts: private nuisance

1(b). Torts: permanent injunction

2(a). Torts: trespass

2(b). Torts: damages

3. Civil Procedure: affirmative defense waiver

4(a). Evidence: hearsay

4(b). Evidence: party statement exception

iii
Question No. 1

Dan, age 67, is the owner and sole proprietor of Big City Art Gallery, a prestigious art

gallery, located in Big City, Pennsylvania. Dan also owns a valuable personal art collection and

owns Mainline Manor, a large, stately home located on two acres in M County, Pennsylvania,

where he has resided since he purchased the property in 1979 for $250,000. Dan has made only

minimal improvements to Mainline Manor and has never owned any other real estate. Dan’s net

worth is $25 million. He is single. His daughter, Beth, works closely with Dan in managing the

gallery. Dan’s other child, Wes, is an M County physician and is not involved in the business.

Dan is also a hot air balloon enthusiast and spends much of his free time piloting his hot air

balloon and attending hot air balloon conventions.

In May 2017, Dan decided to attempt to fly around the world in his hot air balloon. The

night before he left, he asked his friend Angela to meet him at his home. Upon her arrival, he

asked her to prepare his will for him so that his affairs would be in order if anything happened

while he was away. Angela was a well-regarded commercial lawyer at a large M County law

firm and regularly represented Dan on business matters but had never practiced estate or personal

tax law. Angela said “Look Dan, I’d love to help, but an estate as large as yours will have some

complex probate issues, and you may be looking at 40% federal estate tax rates if you don’t plan

properly. You really should talk to an expert - why don’t you come in and meet with my trust

and estate partner when he gets back next week?” Dan replied “Thanks, but I’m leaving

tomorrow and there’s a chance I could die on this trip. Can you just help me get something in

place for now? I promise I’ll meet your partner when I get back, but this is an emergency!”

Angela reluctantly agreed, and, at Dan’s instruction, drafted a will, which provided, in

pertinent part, as follows:

1
I give my real estate at 5 Main Street, Mainline, PA, which is known as Mainline
Manor, to my son Wes. I give the residue of my estate to Wes and Beth, in equal
shares.

Dan properly signed and dated the will on May 20, 2017. Dan then went to a back room and

returned with a painting, which he put in a crate and gave to Angela, saying “this is my best

piece – it’s an original Picasso and worth about $2 million. Hold this in trust for me and if I die,

give the painting to Beth.” Dan departed on his trip the following day.

On June 25, 2017, Dan’s hot air balloon crashed outside Paris, France, and he suffered

traumatic brain injuries rendering him comatose. Beth arranged to have him returned to the

United States, where he remained hospitalized and in a coma. On July 21, 2017, the M County

Orphans’ Court adjudicated Dan to be incapacitated and appointed Beth as guardian of his estate

and person. Beth listed Mainline Manor for sale and quickly reached an agreement to sell the

property. The sale closed on October 5, 2017, with Beth signing the deed and closing documents

as Dan’s guardian. The net proceeds of the sale were two million ($2,000,000) dollars, which

Beth placed into a non-interest bearing account in Dan’s name at Mainline Bank. On January 2,

2018, Dan died. At the time of Dan’s death, Beth had not expended any of the proceeds of the

sale of Mainline Manor.

1. After Dan’s death, Beth filed a petition asking the court for a declaratory judgment that
her father set up a valid trust for the Picasso painting, with her as the beneficiary. Under
the Pennsylvania Probate, Estates and Fiduciaries Code, how will the court rule on the
question of whether Dan set up a valid and enforceable trust?

2. Assume for this question only that Dan’s will was valid and was admitted to probate, and
that the account holding only the two million dollars from the sale of Mainline Manor
was the sole asset in Dan’s probate estate. Who will take under Dan’s will, and in what
amount?

3. What, if any, are the 2017 federal income tax consequences to Dan of the sale of
Mainline Manor? Assume Dan is a cash basis, calendar year tax filer.

4. Did Angela violate any of the Pennsylvania Rules of Professional Conduct by preparing
Dan’s will?

2
Question No. 1: Examiner’s Analysis

1. The Court will deny Beth’s petition because oral trusts are not valid in
Pennsylvania.

“Oral trusts are unenforceable in this Commonwealth.” 20 Pa.C.S.A. § 7737. In


Pennsylvania, a trust may be created by:

(1) transfer of property under a written instrument to another person as trustee


during the settlor’s lifetime or by will or other written disposition taking effect
upon the settlor’s death;
(2) written declaration, signed by or on behalf and at the direction of the
owner of property as required by section 7732 (relating to requirements for
creation — UTC 402), that the owner holds identifiable property as trustee; or
(3) written exercise of a power of appointment in favor of a trustee.

20 Pa.C.S.A. § 7731. Section 7732 requires the following elements:

(1) the settlor has capacity to create a trust;


(2) the settlor signs a writing that indicates an intention to create the trust and
contains provisions of the trust;
(3) the trust has a definite beneficiary or is:
(i) a charitable trust;
(ii) a trust for the care of an animal, as provided in section 7738 . . . ; or
(iii) a trust for a noncharitable purpose, as provided in section 7739
(relating to noncharitable trust without ascertainable beneficiary . . . );
(4) the trustee has duties to perform; and
(5) the same person is not the sole trustee and sole beneficiary of the trust.

20 Pa.C.S.A. § 7732. Pennsylvania law does not require a formal, signed trust instrument or the
use of the words "trust" or "trustee." Peters Creek United Presbyterian Church v. Wash.
Presbytery, 90 A.3d 95, 113 (Pa. Commw. Ct. 2014). A "trust document" is not required, but
there must be "a writing that indicates an intention to create the trust and contains provisions of
the trust." In re Estate of Thomas, 39 Pa. D. & C.5th 103, 111 (C.P. 2014).

It appears from the facts that Dan intended to create a trust. There is no indication in the
facts that Dan lacks capacity. He gave property (the Picasso painting) to Angela, and instructed
her to hold it in trust, thus, transferring the property to the trust. Dan and Beth are named as
beneficiaries of the trust. Angela is appointed as trustee and has duties to perform; she is to hold
the painting and to transfer it to Beth if Dan dies. With Angela as trustee and both Dan and Beth
as beneficiaries, the same person is not sole trustee and sole beneficiary.

However, Dan’s instructions were oral, and he did not sign any writing indicating his
intentions to create a trust and containing the provisions of the trust. Pennsylvania law requires
trusts to be in writing, see 20 Pa.C.S.A. §§ 7731, 7732, and explicitly states that oral trusts will
not be enforced. 20 Pa.C.S.A. § 7737. While a formal trust document is not necessary, some
form of writing is required. Dan’s apparent attempt to create a trust was not in writing and so it
would be invalid and unenforceable.

3
Therefore, the court will determine that despite Dan’s intent, he did not create a valid and
enforceable trust.

2. Wes should receive all of the proceeds of the sale of Mainline Manor because he was
named beneficiary of a specific devise and the gift was not adeemed because Dan
was adjudicated an incapacitated person.

The general rule is a gift of specific property by will that is not contained in the
decedent’s estate at death is adeemed.

It frequently happens that property specifically devised in a will is not owned by


the testator at his death, usually because after execution of the will he sold or
disposed of the property to another. As a general rule, Pennsylvania, like most
American jurisdictions, has regarded the gift as adeemed and does not recognize a
right in the beneficiary to the proceeds of the sale or to property received in
exchange.

Estate of Taylor, 480 Pa. 488, 495, 391 A.2d 991, 994 (1978) (footnote omitted). As to when a
gift by will is a specific devise subject to ademption:

A specific bequest or devise is a gift by will of a specific article or other property,


real or personal, which is identified and distinguished from all other things of the
same kind, and may be satisfied only by delivery of the particular thing.

In re Fox’s Estate, 494 Pa. 584, 587, 431 A.2d 1008, 1010 (1981).

The gift of Mainline Manor to Wes was a specific devise because it was a gift by will of a
specific parcel of real property. Dan’s will referenced the property by name and address.

Because Mainline Manor was sold prior to Dan’s death, it was not a part of his estate.
Accordingly, the devise would be subject to ademption under the common law.

However, the Probate, Estate and Fiduciaries Code contains certain exceptions to the
ademption rule, including the following:

Nonademption; incapacity. -- If property of an adjudicated incapacitated person


specifically devised or bequeathed is sold or exchanged or if a condemnation
award or insurance proceeds are paid to the estate of an incapacitated person as a
result of condemnation, fire or casualty, the specific legatee or devisee has the
right to the net sale price, the property received in exchange, the condemnation
award or the insurance proceeds. This paragraph does not apply if subsequent to
the sale, exchange, condemnation, or casualty, the testator has been adjudicated
not to be an incapacitated person and survives the adjudication by one year.

20 Pa.C.S.A. § 2514(16.1).

4
Here, Mainline Manor was sold by Beth as Dan’s guardian, after he had been adjudicated
incapacitated. At no time thereafter was Dan adjudicated not to be incapacitated, and in fact he
died a short time later. As a result, the nonademption rule applies, and Wes is entitled to receive
the entirety of the proceeds of the sale of Mainline Manor that are contained in the Mainline
Bank account.

3. The federal income tax consequences to Dan resulting from the sale of Mainline
Manor will be taxable income of $1,500,000.

Gross income includes “[g]ains derived from dealings in property.” 26 U.S.C.S. §


61(a)(3). The “gain from the sale or other disposition of property shall be the excess of the
amount realized therefrom over the adjusted basis.” 26 U.S.C.S. § 1001(a). The adjusted basis is
the cost of the property, adjusted over time for factors including capital improvements. The
basis of property is the cost of the property. See 26 U.S.C.S. §§ 1011; 1012(a); 1016.1

Applying the above, the amount realized on the sale of Mainline Manor is $2,000,000.
Dan’s basis in the property is the amount he paid for it, $250,000. Subtracting the adjusted basis
of $250,000 from the $2,000,000 amount realized, there is a gain of $1,750,000. Therefore, Dan
has a potential gain of $1,750,000 on the sale of Mainline Manor.

This does not end the analysis, however, because the Internal Revenue Code contains a
special exclusion for the sale of a principal residence. That rule provides in relevant part as
follows:

(a) Exclusion. -- Gross income shall not include gain from the sale or exchange
of property if, during the 5-year period ending on the date of the sale or exchange,
such property has been owned and used by the taxpayer as the taxpayer's principal
residence for periods aggregating 2 years or more.

(b) Limitations.

(1) In general. -- The amount of gain excluded from gross income under
subsection (a) with respect to any sale or exchange shall not exceed
$250,000.

***

(3) Application to only 1 sale or exchange every 2 years. -- Subsection


(a) shall not apply to any sale or exchange by the taxpayer if, during the 2-
year period ending on the date of such sale or exchange, there was any
other sale or exchange by the taxpayer to which subsection (a) applied.

26 U.S.C.S. § 121.

1
As the question states that Dan has made only minimal improvements to the property there is no need to undertake
a detailed discussion of adjusted basis under 26 U.S.C.S. 1016 et al.

5
Here, Dan has met all of the elements necessary to claim the principal residence
exclusion. He has owned and resided at the residence for at least 2 of the last 5 years; he has
resided there since 1979. Dan has not made use of the exclusion in the past 2 years; in fact, the
question states that he has never owned any other real estate. Accordingly, the principal
residence exception will apply to the sale of Mainline Manor and Dan will be able to exclude an
additional $250,000 from his taxable income.2 From Dan’s amount realized of $2,000,000, he
can subtract his basis of $250,000 which results in a gain of $1,750,000 to be included in gross
income. Dan may exclude $250,000 of that gain because Mainline Manor qualifies as his
personal residence, which will leave a taxable gain of $1,500,000. As a result, Dan will have
taxable income of $1,500,000 for 2017 arising from the sale of Mainline Manor.

4. Angela likely complied with the Rules of Professional Conduct when she prepared
Dan’s will.

The Pennsylvania Rules of Professional Conduct provide in relevant part the following:

A lawyer shall provide competent representation to a client. Competent


representation requires the legal knowledge, skill, thoroughness and preparation
reasonably necessary for the representation.

Pa. RPC 1.1. The comments elaborate on the degree of technical skill required to undertake
representation as follows:

In determining whether a lawyer employs the requisite knowledge and skill in a


particular matter, relevant factors include the relative complexity and specialized
nature of the matter, the lawyer's general experience, the lawyer's training and
experience in the field in question, the preparation and study the lawyer is able to
give the matter and whether it is feasible to refer the matter to, or associate or
consult with, a lawyer of established competence in the field in question. In many
instances, the required proficiency is that of a general practitioner. Expertise in a
particular field of law may be required in some circumstances.

Pa. RPC 1.1, cmt 1. Angela appears to have felt that she lacked the training and experience
necessary to represent Dan in connection with drafting his will. She specifically told Dan that it
was not her area of expertise and suggested he meet with her partner who specializes in estate
and trust law.

While the comments provide that in many cases the proficiency of a general practitioner
is all that is required, Angela herself acknowledged that Dan’s estate was complicated and
required expert counsel. As such, an argument could be made that Angela admitted she did not
possess the knowledge necessary to provide competent representation.

However, the comments to rule 1.1 also provide the following:


In an emergency a lawyer may give advice or assistance in a matter in which the
lawyer does not have the skill ordinarily required where referral to or consultation
or association with another lawyer would be impracticable. Even in an

2
Because Dan is single, no discussion of the rules for joint returns is included here.

6
emergency, however, assistance should be limited to that reasonably necessary in
the circumstances, for ill considered action under emergency conditions can
jeopardize the client's interest.

Pa. RPC 1.1, cmt 3. If as Dan indicated, his imminent and potentially dangerous travel plans
constituted an emergency, then Angela may have acted properly in preparing his will as it may
have been reasonably necessary under the circumstances. Moreover, commentators have
recognized that in estate planning an enhanced risk of death may create an emergency:

Insofar as consistent with providing the client with competent representation, the
lawyer should adhere to the established schedule and inform the client of any
revisions that are required, whether attributable to the lawyer or to circumstances
beyond the lawyer’s control. Many clients engaged in estate planning are elderly
or are facing medical emergencies. There is thus an enhanced risk that the client
might die or otherwise become incapable of completing an estate plan if the estate
planner takes more time than is reasonable under the circumstances to do the
work requested. In such cases the client may be harmed, and intended
beneficiaries may not receive the benefits the client intended them to have.

AMERICAN COLLEGE OF TRUST & ESTATE COUNSEL, COMMENTARIES ON THE MODEL


RULES OF PROFESSIONAL CONDUCT at 57 (5th ed. 2016).

Under the facts and as limited by the question, Dan impressed upon Angela the increased
likelihood of his death and his immediate need for estate planning to ensure his beneficiaries
received the property he intended them to have. He promised to consult with an expert upon his
return, meaning that Angela’s representation would have been limited to ensuring Dan had a will
in place during the time he was out of the country and engaged in dangerous travel activity.
Therefore, it seems reasonable under the circumstances for Angela to have treated Dan’s
situation as an emergency.

7
Question No. 1: Grading Guidelines

1. Oral trust.

Comments: Applicants should discuss the requirements to establish a valid trust and, in
particular, recognize that oral trusts are not enforceable under Pennsylvania Probate, Estates and
Fiduciaries law.

4 points

2. Ademption.

Comments: Applicants should recognize that a specific devise is adeemed if not contained in
testator’s estate at death and discuss the exception if the property was sold while the testator was
incapacitated and determine that the bequest of Mainline Manor was not adeemed.

6 points

3. Capital gain on sale of personal residence.

Comments: Applicants should analyze the computation of taxable gain using cost basis, and
discuss the application of the principal residence exception and conclude that Dan had income of
$1,500,000 for his 2017 federal income taxes.

5 points

4. Competency of lawyer.

Comments: Applicants should identify the issue of Angela’s competency to undertake the
representation and come to a well-reasoned conclusion on whether or not Angela violated the
rules of professional conduct.

5 points

8
Question No. 2

Several years ago, Al, Ben, and Carl, who all had excavating experience, formed Earth

Movers (“EM”), a Pennsylvania general partnership. They have been and continue to be EM’s

sole partners. EM is based in C City, Pennsylvania, which borders State X. EM’s written

partnership agreement prohibits any partner from committing EM to a contract in excess of

$5,000 without the consent of all partners.

C City has grant programs to support small businesses located in C City. One program

offers a $50,000 grant to C City businesses that have been in existence for at least one year. Al

obtained a grant application on behalf of EM. Grant guidelines require that all owners of an

applicant have a domicile in Pennsylvania. Al and Ben are lifetime Pennsylvania residents. Carl

has always lived with his wife in State X. Al, Ben, and Carl discussed the application and Carl

agreed to rent a one room apartment in C City. EM prepared the grant application and listed

Carl’s address as the C City apartment. C City always follows up to verify the domicile of each

applicant’s owners. C City’s investigation has revealed that Carl sleeps at his home with his

wife in State X nearly every night, has a State X driver’s license, recently voted in an election in

State X, and has barely furnished the apartment in C City.

Last month, Ben was working on a residential sewer line project that EM had obtained a

contract to complete. Even though he knew that standard procedure employed by excavators and

state guidelines required that he should not begin excavation without verification of underground

utility lines, he was in a hurry so he began excavation without the underground lines being

marked. While excavating, Ben broke a water line leading to a neighboring home which caused

extensive flooding of the basement of the home. Had Ben waited for the lines to be marked he

would have known the location of the line. There is no question that Ben’s actions were

negligent.

9
Two weeks ago, Ben contacted Trailers, Inc. (“TI”), a local company that manufactures

and sells custom trailers. Ben knew TI’s president, Pete, socially. Pete knew that Ben was a

partner in EM and had ordered parts for EM from TI in the past. Pete had never seen EM’s

partnership agreement. Ben told Pete that EM needed a custom-built trailer that would

accommodate EM’s excavating equipment with features not available on stock trailers. Pete told

Ben that he could design and manufacture the trailer that EM needed but that if EM did not buy

the trailer he would not be able to sell it in the ordinary course of TI’s business. Ben and Pete

orally agreed that TI would build the trailer for EM and EM would buy it when completed for

$8,000. The next day, TI purchased the materials that it needed to build the trailer and began its

production. As of yesterday, the trailer was 75% complete.

Al and Carl are fed up with Ben’s lax attitude about following operational rules. When

they confronted Ben about the damages to the home, he laughed and told them that he had also

ordered a custom trailer from TI.

1. Assume that the domicile requirement in C City’s grant guidelines is


constitutional. Under the domicile guidelines, is EM eligible to receive the C City
grant?

2. If the homeowner whose house was damaged by Ben’s actions sues Al, Ben, Carl,
and EM, are Al, Ben, Carl, and/or EM liable for damages as a result of Ben’s
negligent act?

3. Al intends to contact TI’s president today and tell him that EM will not honor the
agreement made by Ben because it was oral. Under the Uniform Commercial
Code, does TI have any basis to enforce the agreement?

4. Assume for this question that the fact that the agreement between EM and TI was
oral is not a defense to the enforceability of the agreement. Will EM be
successful in asserting a defense against TI if TI seeks to enforce the agreement
based on the language in EM’s partnership agreement requiring the approval of all
partners for contracts in excess of $5,000?

10
Question No. 2: Examiner’s Analysis

1. EM will not be eligible to receive the grant because Carl has not established
domicile in Pennsylvania.

To be eligible for the $50,000 grant EM must be able to show C City that all of its
owners, i.e., its general partners, maintain domicile in the Commonwealth of Pennsylvania. Since
the grant is administered in Pennsylvania, Pennsylvania law should be applied to determine the
question of domicile. Greenwood v. Hildebrand, 357 Pa. Super. 253, 258, 515 A.2d 963, 965
(1986). It is clear that Al and Ben maintain their domiciles in Pennsylvania but Carl’s domicile
is in question.

The determination of domicile is driven by the facts of each case. Civil Service Comm’n
of Pittsburgh v. Parks, 80 Pa. Cmwlth. 134, 136, 471 A.2d 154, 156 (1984).

A domicile is the place at which an individual has fixed his family home and
principal establishment for an indefinite period of time. A domicile once acquired
is presumed to continue until it is shown to have been changed and where a
change is alleged, the burden of proving it rests upon whoever makes the
allegation. A new domicile can be acquired only by physical presence at a new
residence plus intent to make that new residence the principal home. Intent is the
actual state of facts, not what one declares them to be.

In Re Prendergast, 543 Pa. 498, 506, 673 A.2d 324, 327-28 (1996) (citations omitted). Although
one may have multiple residences, one can have only one domicile. See Greenwood, 357 Pa.
Super. at 259, 515 A.2d at 965; Restatement (Second) Conflict of Laws § 11(2) (Am. Law Inst.
1971). Thus, the question of domicile is a matter of intention, unlike residence, which is a
physical fact. See Greenwood, 357 Pa. Super. at 259, 515 A.2d at 965.

Carl has arguably established a residence in C City by renting the apartment. He has also
declared it to be his residence in the grant application filed with C City. The courts, however,
have made it clear that his stated declaration is not determinative. Instead, one must look to the
intent demonstrated by his acts and conduct. Civil Service Comm’n of Pittsburgh v. Parks, 80 Pa.
Cmwlth. at 138, 471 A.2d at 157. Even though Carl has an apartment in C City, he really does
not live in the apartment. Carl sleeps at home with his wife nearly every night, has a State X
driver’s license, recently voted in an election in State X, and has barely furnished the apartment
in C City. Clearly, Carl has not exhibited conduct demonstrating an intent to have C City
become his permanent home. To the contrary, his actions support a finding that he intends State
X to be his permanent home and, accordingly, his domicile. On this basis C City should find
Carl’s domicile to be State X and deny EM’s request for the grant.

2. EM and all the partners of EM are liable for the damages caused by the wrongful
act committed by Ben.

The Pennsylvania Uniform Partnership Act of 2016 (the “Act”), addresses the liability of
partners and a partnership for the wrong act of a partner. Under the Act both the partnership and
the partners in the partnership will have liability for the damages caused by Ben’s negligent act.
Section 8435 of the Act provides “[a] partnership is liable for loss or injury caused to a person,

11
or for a penalty incurred, as a result of a wrongful act or other actionable conduct, of a partner
acting in the ordinary course of business of the partnership or with the actual or apparent
authority of the partnership.” 15 Pa. C.S.A. §8435.1 Section 8435 basically follows the agency
law doctrine of respondeat superior. To successfully proceed under this Section a plaintiff must
show (i) wrongful conduct by a general partner, (ii) that caused a loss or injury, and (iii) at all
relevant times the general partner was acting with actual or apparent authority or within the
ordinary course of business of the partnership. 15 Pa. C.S.A.§8435 cmt..

The Act further provides: “Except as provided in subsection (b) [dealing with preexisting
liabilities] or section 8204 . . . [dealing with limitations on liability of a partner in a limited
liability partnership or limited liability limited partnership], all partners are jointly and severally
liable for all debts, obligations and other liabilities of the partnership unless otherwise agreed by
the claimant or provided by law.” 15 Pa. C.S.A. §8436(a). This section generally follows
former section 8327 but expands joint and several liability to both tort and contract based
claims.2

Thus, a partnership is liable for damages caused by the negligence of one of its partners if
the partner committed the act while acting in the ordinary course of and in furtherance of the
partnership’s business. In this case, Ben caused damages to the owner of a property when he
broke a water line leading to the property owner’s home. Had Ben followed proper procedure
and had the underground lines located prior to commencing excavation he most likely would not
have ruptured the line. He was in a hurry and did not follow proper procedure normally followed
by excavators; thus, acting negligently. When Ben broke the water line he was engaged in the
act of excavating for a customer of EM, i.e. in the course of EM’s business. Accordingly, EM
will be liable to the owner of property for the water damage. Additionally, subject to the
limitations of the act, all partners are answerable jointly and severally for the claim chargeable to
the partnership. Therefore, the owner of property may also be able to look to the individual
partners of EM jointly and severally to recover if the partnership does not or cannot satisfy its
claim.

3. The oral agreement should be enforceable under the Pennsylvania Uniform


Commercial Code (the “Code”) as one involving specially manufactured goods.

Section 2102 of the Code indicates that the Code applies to transactions in goods. 13
Pa.C.S.A. § 2102. “‘Goods’ means all things (including specially manufactured goods) which
are movable at the time of identification to the contract for sale other than the money in which
the price is to be paid, investment securities (Division 8) and things in action.” 15 Pa. C.S.A. §

1
This Section is derived from former Section 8325 which provided: Where by any wrongful act or
omission of any partner acting in the ordinary course of the business of the partnership or with the authority of his
copartners, loss or injury is caused to any person, not being a partner in the partnership, or any penalty is incurred,
the partnership is liable therefore to the same extent as the partner so acting or omitting to act. 15 Pa. C.S.A.§8325
(repealed April 1, 2017).

2
Note: Section 8437(d) of the Act requires partnership creditors to exhaust the partnership’s assets before
levying on a judgment debtor partner’s individual property where the partner is personally liable for partnership
obligation under Section 8436 of the Act. “The rule in subsection (d) respects the concept of the partnership as an
entity and makes partners more in the nature of guarantors than principal debtors on every partnership debt.”
Committee Comment (2016), subsection (d). 15 Pa. C.S.A.§8437.

12
2105. Thus, the agreement for the trailer to be manufactured by TI for sale to EM is a
transaction in goods under the Code. Section 2201 of the Code generally requires a contract for
the sale of goods for a price of $500 or more to be in writing and signed by the party to be
charged in order for the contract to be enforceable. 13 Pa. C.S.A. § 2201(a). Here, we have a
completely oral contract which ostensibly would be unenforceable under this statute of frauds
provision. The Code does, however, provide an exception to the writing requirement where
goods have been specially manufactured and are not suitable for resale by the seller in the
ordinary course of its business. 13 Pa. C.S.A. § 2201(c)(1). Thus, the Code recognizes the fact
that seller’s do not produce custom-made goods just for the exercise. White and Summers,
Uniform Commercial Code, § 2-5 (4th Ed. 1995). This would not be good business and would
most likely lead to losses or bankruptcy.

For the specially manufactured goods exception to the statute of frauds to apply five
elements must be established by the party seeking enforcement of the contract. The elements are:

(1) The goods must have been specially manufactured for the buyer.

(2) The goods, once manufactured, must be goods not suitable for sale to other
persons in the seller’s ordinary course of business.

(3) The seller must have made a substantial start in the manufacture of the goods.

(4) The seller’s beginning of manufacture occurred under circumstances that


reasonably indicate the goods are for the buyer.

(5) The actions of the seller occurred before the seller received any notice of
repudiation.

See, generally, White and Summers, Uniform Commercial Code, § 2-5 (4th Ed. 1995); Quinn’s
Uniform Commercial Code Commentary and Law Digest, Vol. 1, 2nd Ed., ¶ 2-201[A][8].

TI should have no problem establishing each of the foregoing elements, thus taking the
contract out of the statute of frauds. The trailer is being specially manufactured for the buyer, is
not suitable for resale by TI, and the trailer was 75% completed before Al told TI that EM would
not honor the verbal agreement. TI’s start of manufacturing of the trailer was done so solely for
the benefit of EM. Also, TI completed 75% of the work on the trailer prior to Al advising TI that
EM did not want the trailer. Therefore, the specially manufactured goods exception set forth in
the Code should apply and the contract should be enforceable.

4. EM will not be successful in raising the provision contained in the partnership


agreement as a defense to TI’s action to enforce the contract.

The Act provides that the partnership agreement by and among the partners will govern
the “relations among the partners as partners and between the partners and the partnership.” 15
Pa. C.S.A. § 8415(a)(1). Thus, the limitation on contracting set forth in the partnership
agreement is enforceable by the partners. The question, however, is whether or not the limitation
is enforceable as to a third party who acted without knowledge of the limitation contained in the
partnership agreement.

13
In this regard, the Act provides:

Subject to the effect of a certificate of partnership authority under section 8433


(relating to certificate of partnership authority) [which is not applicable in this
case], the following rules apply:

(1) Each partner is an agent of the partnership for the purpose of its business. An
act of a partner, including the signing of an instrument in the partnership name,
for apparently carrying on in the ordinary course the partnership business or
business of the kind carried on by the partnership binds the partnership, unless the
partner did not have authority to act for the partnership in the particular matter
and the person with which the partner was dealing knew or had notice that the
partner lacked authority.

(2) An act of a partner which is not apparently for carrying on in the ordinary
course the partnership’s business or business of the kind carried on by the
partnership binds the partnership only if the partner had actual authority to take
the action.

15 Pa. C.S.A. § 8431.3 This section essentially follows the general common law principal that a
general partner was considered a general agent of the partnership. 15 Pa. C.S.A. § 8431 cmt.

Thus, under section 8431 of the Act a third person dealing with a partner apparently
carrying on the business of the partnership is not affected by restrictions on the partner’s
authority contained in the partnership agreement if the third person has no notice or knowledge
of the restrictions. Under the above facts, Pete knew that Ben was an EM partner and had acted
on behalf of EM in the past. Although Pete did not know if Ben had express authority to act on
behalf of EM when ordering the trailer he had a reasonable basis to believe that Ben had at least
apparent authority to do so. Also, Pete did not know of the restriction in the EM partnership
agreement. Since Ben was acting on what appeared to be partnership business, the agreement
with TI would bind EM despite the restriction in the partnership agreement and Ben’s lack of
express authority in the matter.

3
This section essentially tracks former section 8321.

14
Question No. 2: Grading Guidelines

1. Conflict of Laws – Domicile Issue.

Comments: The candidates should discuss domicile and the issues to be analyzed in a domicile
determination and apply that analysis to the facts presented.

5 points

2. Partnerships – Liability for wrongful act of partner

Comments: The candidates should recognize that the partner who committed the wrongful act
was acting on partnership business and discuss the liability of the partnership and the other
partners for damages caused by the wrongful act.

5 points

3. Sales – Specially Manufactured Goods Exception to Statute of Frauds

Comments: The candidates should discuss the elements of the specially manufactured goods
exception to the statute of frauds in the context of the facts presented.

5 points

4. Partnerships – Effect of limitation in partnership agreement upon contract with


third party.

Comments: The candidates should discuss the effect a limitation in the partnership agreement
has on a contract entered into with the third party when that limitation is not known by a third
party.

5 points

15
Question No. 3

Jeff, 23, and Paige, 21, lifelong residents of C County, Pennsylvania, have dated for

several years but have not had sex. Unbeknownst to Paige, Jeff is incurably impotent (unable to

have sexual intercourse), which he has always known but never told Paige.

Neither Jeff nor Paige is employed, which has prevented them from being able to finance

a wedding. In June of 2017, Jeff unilaterally decided to take matters into his own hands to

arrange the marriage ceremony for the couple and get some spending money. Jeff went to a

parish rectory (residence) in Pennsylvania, about 40 miles away from C County, and took Pastor

Ian from the rectory at gunpoint. Jeff’s plan was to have Pastor Ian marry Jeff and Paige, and

Jeff would then release the pastor after getting some ransom money. Jeff directed Pastor Ian into

his vehicle and drove him to Paige's apartment located in an isolated area of C County,

Pennsylvania. Once at Paige’s apartment, Jeff told Paige that he was here with the pastor so they

could get married. Although Paige was extremely surprised about the unexpected arrival of

Pastor Ian, she was eager to complete the marriage to formally start her life together with Jeff.

Paige participated in the ceremony and completed the marital vows resulting in the marriage’s

taking place.

Upon completion of the ceremony, Jeff called Pastor Ian’s secretary and asked for

$10,000 in ransom to be delivered to Paige’s apartment. Jeff told the secretary that, once the

ransom was delivered, he would release the pastor. Instead of securing the ransom, the secretary

called the C County Police. Officer Rachel, who was in full uniform, responded to the call and

approached Paige’s apartment door from the outside. Jeff pointed his gun out the window of the

apartment directly at Officer Rachel’s leg and fired a shot in an attempt to slow her down but

without any intention of seriously injuring her. The shot grazed Officer Rachel’s leg causing a

16
superficial wound, i.e. bodily injury. She fled the scene in significant pain. Two hours after Jeff

shot Officer Rachel, other officers stormed the apartment and arrested Jeff. Pastor Ian, who had

been under Jeff’s control for nearly five hours at that point, was uninjured.

1. What arguments under the facts above could the Commonwealth make to support
kidnapping charges against Jeff regarding the actions he took regarding Pastor
Ian?

2. Would aggravated assault charges be supported against Jeff regarding the actions
taken against Officer Rachel?

Three weeks after Jeff’s arrest, Paige learns that Jeff is impotent. She becomes enraged

by this fact and feels that she can no longer trust Jeff.

3. Assuming that all other steps were taken to form a legal marriage, what type of
action, other than declaratory judgment, could Paige file in C County seeking to
have her marriage to Jeff declared void by the court based upon his impotence,
and how would that action be commenced?

Immediately after his arrest for the kidnapping and aggravated assault charges, Jeff was

lodged in the C County jail. As part of the intake procedure, Jeff was informed that all written

communications sent or received by inmates were subject to review by prison personnel, and Jeff

confirmed in writing his receipt of that information. Four weeks after his incarceration, he sent a

letter to his wife, Paige, in which he made incriminating statements concerning the kidnapping

charge involving Pastor Ian. When Paige received the letter, she immediately provided it to the

police and informed them that she wished to testify against Jeff at trial. Assume the kidnapping

charge proceeds to trial, Jeff and Paige are still married, and the Commonwealth calls Paige to

testify to the contents of the letter received from Jeff. Jeff directs his counsel to object to Paige

testifying on the basis of the spousal communication privilege.

4. Assuming for the purpose of this question only that the marriage was valid, how
should the court rule on the defense objection to preclude Paige from testifying
against Jeff regarding the contents of the letter sent to her?

17
Question No. 3: Examiner’s Analysis

1. The Commonwealth should argue that a kidnapping charge is supported against


Jeff because he took Pastor Ian a substantial distance and held him for a substantial
period of time in a place of isolation in order to, in part, obtain ransom.

“[A] person is guilty of kidnapping if he unlawfully removes another a substantial


distance under the circumstances from the place where he is found, or if he unlawfully confines
another for a substantial period in a place of isolation” with the intention, inter alia, of holding
the victim for ransom. 18 Pa. C.S.A. § 2901(a)(1). A removal or confinement is unlawful under
subsection (a) of the statute if it is accomplished by force or threat. 18 Pa. C.S.A. § 2901(b)(1).
The Commonwealth should argue under both scenarios, i.e. removal a substantial distance and
isolation for a substantial period of time. In Commonwealth v. Ruehling, 334 A.2d 702, 703 (Pa.
Super. 1975), the court held that 30 miles was a “substantial distance” for the purposes of the
kidnapping statute. The substantial distance element of the kidnapping statute was also found to
have been satisfied where the victim was taken a distance of approximately six miles.
Commonwealth v. Dehoniesto, 624 A.2d 156, 160 (Pa. Super. 1993). Evidence of a one hour
confinement was sufficient to permit a jury to decide whether the substantial period element of
kidnapping was satisfied. Commonwealth v. Hook, 512 A.2d 718, 720 (Pa. Super. 1986). As to
what qualifies as a place of isolation, the Superior Court has held that “the concept is ‘not
geographic isolation, but rather effective isolation from the usual protections of society.’”
Appeal of T.G., 836 A.2d 1003, 1008 (Pa. Super. 2003) quoting Commonwealth v. Mease, 516
A.2d 24, 26 (Pa. Super. 1986). More recently, the Supreme Court has held that “place of
isolation” under the kidnapping statute is not geographic in nature, but contemplates the
confinement of a victim, where he or she is separated from the normal protections of society in a
fashion that makes discovery or rescue unlikely. See Commonwealth v. Rushing, 99 A.3d 416,
425 (Pa. 2014).

As applied here, the Commonwealth should argue under both of the above scenarios. It
should first argue that Jeff removed Pastor Ian a substantial distance from his parish rectory to
Paige’s apartment, which was about forty miles away. The removal was accomplished by Jeff
using a handgun and said force and threat, whether actual or implied, will likely constitute an
unlawful removal under 18 Pa. C.S.A. § 2901(b)(1). This would likely satisfy the “substantial
distance” element of the kidnapping statute as supported by both the Reuhling and Dehoniesto
holdings cited above. Second, the Commonwealth should argue that Pastor Ian was confined for
a substantial period of time in isolation. In particular, the facts indicate that from the time Pastor
Ian was taken by Jeff to the time that Jeff was arrested a period of approximately five hours
passed. Under Hook, cited above, this period of time would be sufficient to support the
“substantial period” element of the kidnapping statute. The Commonwealth would also argue
that Paige’s apartment would be deemed to be a place of isolation as Pastor Ian was clearly being
held in a location where he did not enjoy the usual protections of society because there was no
one around that he could call out to for assistance or rescue as the apartment was located in an
isolated area. Finally, under both scenarios, the Commonwealth should argue that Jeff’s
intentions, in part, were to hold Pastor Ian for ransom. The facts indicate that this was his plan

18
from the start. This is further evidenced by the fact that he contacted Pastor Ian’s secretary and
demanded $10,000 in cash in order to secure the Pastor’s release.

2. Aggravated assault charges would likely be supported by the facts relative to the
actions taken by Jeff against Officer Rachel because he caused bodily injury to the
officer in the performance of her duties and because he caused bodily injury by
using a deadly weapon.

“A person is guilty of aggravated assault if he . . . attempts to cause or intentionally or


knowingly causes bodily injury to any of the officers, agents, employees or other persons
enumerated in subsection (c), in the performance of [their] duty.” 18 Pa. C.S.A. § 2702(a)(3).
“The officers, agents, employees and other persons referred to in subsection (a)(3)” include
police officers. 18 Pa.C.S.A. § 2702(c)(1). Bodily injury is defined as “impairment of physical
condition or substantial pain.” 18 Pa. C.S.A. § 2301. “In a prosecution for aggravated assault on
a [police] officer, the Commonwealth has no obligation to establish that the officer actually
suffered a bodily injury; rather, the Commonwealth must establish only an attempt to inflict
bodily injury, and this intent may be shown by circumstances which reasonably suggest that [the
defendant] intended to cause injury.” Commonwealth v. Mucci, 143 A.3d 399, 409 (Pa. Super.
2016) (internal quotation and citation omitted). Alternatively, a person can be found guilty of
aggravated assault if he “attempts to cause or intentionally or knowingly causes bodily injury to
another with a deadly weapon.” See 18 Pa. C.S.A. § 2702(a)(4).

As applied here, the facts indicate that Officer Rachel responded, in full uniform, to
Paige’s apartment in response to the call from Pastor Ian’s secretary. Rachel was clearly
engaged in the performance of her duties as a police officer when she responded to this call. As
she approached the apartment door, Jeff pointed his handgun out the window and fired a shot at
her leg in an attempt to slow her down. Jeff grazed Officer Rachel’s leg which caused her
significant pain and the facts state that this resulted in bodily injury. Since Jeff caused bodily
injury to Officer Rachel in the performance of her duties an aggravated assault charge under 18
Pa. C.S.A. § 2702(a)(3) is likely supported by the facts. Further, since Jeff used a gun, i.e. a
deadly weapon, in his actions which caused bodily injury to Rachel, a charge under 18 Pa.
C.S.A. § 2702(a)(4) is also likely supported by the facts. It is clear from the facts that Jeff
intended to injure Officer Rachel when he fired the shot at her leg.

Based upon the above, it is likely that a charge of aggravated assault would be supported
under 18 Pa. C.S.A. § 2702(a)(3) or (a)(4) against Jeff relative to the actions taken against
Officer Rachel.

3. An action in annulment should be commenced in the C County Court of Common


Pleas by filing a complaint with the C County Prothonotary.

“The marriage of a person shall be deemed voidable and subject to annulment . . .


including where either party to the marriage was at the time of the marriage and still is naturally
and incurably impotent unless the condition was known to the other party prior to the marriage.”
23 Pa. C.S.A. § 3305(a)(4).

19
In all cases of marriages which are voidable, either party to the marriage may seek
and obtain an annulment of the marriage but, until a decree of annulment is
obtained from a court of competent jurisdiction, the marriage shall be valid. The
validity of a voidable marriage shall not be subject to attack or question by any
person if it is subsequently confirmed by the parties to the marriage or if either
party has died.

23 Pa. C.S.A. § 3305(b). The definition of “impotence” as a cause for obtaining an annulment of
marriage includes incapacity resulting from physical malfunction or impairment of the sexual
organs. Manbeck v. Manbeck, 489 A.2d 748 (Pa. Super. 1985).

Under the annulment provisions of the Pennsylvania Rules of Civil Procedure, Pa.R.C.P.
No. 1920.1(a) “action” is defined as “an action of divorce or an action for annulment of
marriage.” Pa.R.C.P. No. 1920.3 provides that an action shall be commenced by filing a
complaint with the Prothonotary.

As applied here, the facts provide that Jeff and Paige were married by Pastor Ian. The
call of the question states that all other steps were taken to form a legal marriage. However, it is
clear that Paige went through with the marriage without knowledge that Jeff was impotent. Jeff
has been impotent his entire life, and the facts state it is incurable. Jeff did not tell Paige of these
facts and there is no indication that she otherwise knew about it. When Paige found out about it,
the facts provide that she felt she could not trust Jeff any longer and she apparently had no
interest in continuing to be married to Jeff under these circumstances. In order for Paige to have
the court declare her marriage void, she should pursue an annulment due to Jeff’s impotence.
The impotence appears to have been natural, as he had the condition his entire life, and it was
incurable as stated by the facts. These facts will likely provide a strong basis for establishing the
marriage is voidable and an annulment action should be successful. Pursuant to Pa.R.C.P. Nos.
1920.1 (a) and 1920.3 the annulment action should be commenced via the filing of a complaint
with the Prothonotary of the Court of Common Pleas of C County.

4. The court will likely permit Paige to testify against Jeff relative to the contents of
the letter as the court will likely conclude that Jeff did not have a reasonable
expectation of confidentiality in the letter he sent to Paige and the spousal
communication privilege would not likely apply.

“[I]n a criminal proceeding neither husband nor wife shall be competent or permitted to
testify to confidential communications made by one to the other, unless the privilege is waived
upon the trial.” 42 Pa. C.S.A. § 5914.1 “This privilege, which is waivable only by the spouse
asserting the privilege, prevents a husband or wife from testifying against their spouse as to any
communications which were confidential when made and which were made during the marital
relationship.” Commonwealth v. May, 656 A.2d 1335, 1341-42 (Pa. 1995) citing
Commonwealth v. Newman, 633 A.2d 1069, 1072 (1992) (further citation omitted). In May, the
court was faced with a situation where the appellant sent a letter to his wife after he signed a

1
42 Pa. C.S.A. § 5913 provides, with certain exceptions not applicable here, that in a criminal proceeding a person
shall have the privilege, which he or she may waive, not to testify against his or her then lawful spouse. This
provision is not implicated here as Paige wants to testify against Jeff at trial.

20
form which permitted prison officials to review all incoming and outgoing mail. The court
agreed with the Commonwealth’s argument that the appellant had no reasonable expectation that
his communications to his wife would be held confidential in these circumstances. The May
Court, citing Commonwealth v. Darush, 420 A.2d 1071 (Pa. Super. 1980) (overruled on other
grounds in Commonwealth v. Darush, 459 A.2d 727 (Pa. 1983), stated that “[i]n order for the
spousal privilege rule to apply, it is essential that the communication be made in confidence and
with the intention that it not be divulged.” May, 656 A.2d at 1342. The May Court went on to
conclude that it “d[id] not believe that Appellant could have possessed a reasonable expectation
that the information contained in his correspondence to his wife would remain confidential when
prison officials had the right to read all of his letters.” May, 656 A.2d at 1342. Confidential
communications protected under § 5914 have been limited to oral or written communications.
Commonwealth v. McBurrows, 779 A.2d 509, 514 (Pa. Super. 2001); see also Commonwealth v.
May, 656 A.2d 1335, 1342 (Pa. 1995).

As applied here, Jeff and Paige were married, which would normally provide Jeff with
the ability to assert the spousal communication privilege to prevent Paige from testifying as to
any confidential communications made between the two of them. However, Jeff was informed,
and acknowledged in writing, that all of his letters were subject to review by prison officials.
Accordingly, Jeff should not have had a reasonable expectation of privacy in the letters he was
sending to his wife as he clearly acknowledged that they could be reviewed by third parties,
namely the prison officials. Therefore, it is likely that the spousal communication privilege
would not be applicable and the court would likely permit Paige to testify to the contents of the
letter she received from Jeff.

21
Question No. 3: Grading Guidelines

1. Kidnapping

The Commonwealth should argue that kidnapping charges are supported against Jeff because he
took Pastor Ian a substantial distance and held him for a substantial period of time in a place of
isolation in order to, in part, obtain ransom.

6 Points

2. Aggravated Assault

Aggravated assault charges will likely be supported by the facts relative to the actions taken by
Jeff against Officer Rachel because he caused bodily injury to the officer in the performance of
her duties and because he caused bodily injury by using a deadly weapon.

5 Points

3. Annulment

An action in annulment should be filed on the grounds of Jeff’s impotence and the action should
be commenced in the C County Court of Common Pleas by filing a Complaint.

5 Points

4. Spousal Communication Privilege

The court will likely permit Paige to testify against Jeff relative to the contents of the letter as the
court will likely conclude that Jeff did not have a reasonable expectation of confidentiality in the
letter and the spousal communication privilege would not likely apply.

4 Points

22
Question No. 4

Doc is a 66-year-old physician who is respected by his colleagues and has never been the

subject of a malpractice or disciplinary action. For 10 years, Doc was employed by Huge

Medical (HugeMed), which employs thousands of medical professionals in State A, but does not

do business in any other states. Prior to working for HugeMed, Doc was licensed to practice in

State B, but he allowed his State B license to lapse because there is no HugeMed office within

200 miles of State B. Eighteen months ago, Doc sent an email from his HugeMed email account

inquiring about reinstatement of his State B license. In order to reinstate a lapsed license, a

statute in State B (Reinstatement Act), requires non-resident physicians to pay a $5,000

reinstatement fee and complete 50 hours of professional education, while physicians who reside

within State B have to pay a $500 reinstatement fee and take 10 hours of professional education.

The Reinstatement Act’s legislative history indicates that the reason for treating non-

resident physicians differently is a belief that resident physicians are more likely to provide the

highest quality of care to patients in their own communities. Resident and non-resident

physicians are subject to the same regulations and laws regarding the standard of care, and

statistics show that historically and proportionally there are 20% more disciplinary actions and

30% more malpractice claims brought against resident physicians in State B than there are

against non-resident physicians.

HugeMed monitors employee emails daily, and HugeMed’s information technology

personnel immediately reported Doc’s email to management at HugeMed, including to Doc’s

supervising physician, Phil. As Doc’s supervising physician, Phil had the authority to terminate

Doc’s employment. Nine months ago, Phil met with Doc and told him that HugeMed had

discovered Doc’s email inquiry about reinstatement of his State B license. Doc explained that he

23
is considering moving to State B in five years to be closer to family, but plans to stay at

HugeMed until then. Notwithstanding, Phil informed Doc that HugeMed was terminating Doc’s

employment immediately. Phil said, “We would keep you as long as possible, but we can’t risk

your staying to lure patients away.”

HugeMed offered Doc six months of severance pay only if Doc would execute a

severance agreement (Agreement) that Phil provided to Doc during the meeting. The Agreement

included the following language: “I hereby release, waive and forever discharge HugeMed from

all actions, including actions brought pursuant to the Age Discrimination in Employment Act. I

understand that this release is immediately and irrevocably binding upon execution.” Phil told

Doc that if he wanted severance pay, Doc had to sign the Agreement before leaving the office

that same day. Doc signed the Agreement, packed his personal items and left the office. Ten

days later, and before he received any of his severance pay pursuant to the Agreement, Doc

learned that HugeMed had hired a 46-year-old doctor to replace him.

1. Doc brings an action against the proper parties in the appropriate federal district
court challenging the constitutionality of State B’s Reinstatement Act under the
Privileges and Immunities Clause of the United States Constitution. What is the
likelihood that Doc’s challenge will be successful?

2. Having exhausted his administrative remedies, Doc also brings a federal lawsuit
against HugeMed for age discrimination under the Age Discrimination in
Employment Act (ADEA). If Doc argues that the waiver of ADEA claims in the
Agreement is defective pursuant to the Older Workers Benefit Protection Act
(OWBPA) amendment to the ADEA, what is the likelihood that the court will rule
in Doc’s favor on this issue?

3. Assume for this question only, that the court ruled in Doc’s favor on the waiver
issue. In the ADEA lawsuit, all of the facts set forth above are admissible
evidence of record at the conclusion of discovery, and HugeMed filed a timely
motion for summary judgment on Doc’s ADEA claim. Doc concedes that he has
no direct evidence of age discrimination, but argues that HugeMed’s proffered
reason for terminating his employment was a pretext for age discrimination.
Applying both the applicable procedural and substantive legal standards, how is
the court likely to rule on HugeMed’s motion for summary judgment?

24
Question No. 4: Examiner’s Analysis

1. Doc Will Likely Be Successful in Challenging the Reinstatement Law Under the
Privileges and Immunities Clause of the United States Constitution

Article IV, Section 2, Clause 1 of the United States Constitution, the Privileges and
Immunities Clause, provides, “[t]he Citizens of each State shall be entitled to all Privileges and
Immunities of Citizens in the several States.” The purpose of the Privileges and Immunities
Clause is to “strongly . . . constitute the citizens of the United States [as] one people, by placing
the citizens of each State upon the same footing with citizens of other States, so far as the
advantages resulting from citizenship in those States are concerned.” McBurney v. Young, 133
S.Ct. 1709, 1714 (2013) (internal quotations and citation omitted).

Supreme Court precedent requires a two-step inquiry in examining a challenge under the
Privileges and Immunities Clause. Supreme Court of Virginia v. Friedman, 487 U.S. 59, 64
(1988). First, the restricted activity must be “sufficiently basic to the livelihood of the Nation . . .
as to fall within the purview of the Privileges and Immunities Clause. . . .” Id. at 64 (internal
quotations and citation omitted). Second, even if a challenged restriction deprives nonresidents
of a protected privilege, it should be invalidated “only if [the Court] conclude[s] that the
restriction is not closely related to the advancement of a substantial state interest.” Id. at 65.

Regarding the first part of the two-step inquiry outlined by the Supreme Court, not all
discrimination against nonresidents of a state violate the Privileges and Immunities Clause.
United Bldg. & Constr. Trades Council v. Camden, 465 U.S. 208, 218 (1984). Rather, “[o]nly
with respect to those ‘privileges’ and ‘immunities’ bearing upon the vitality of the Nation as a
single entity must the State treat all citizens, resident and nonresident, equally.” Id. In sum, the
Supreme Court requires an activity to be “fundamental to the promotion of interstate harmony so
as to fall within the purview of the Privileges and Immunities Clause.” Id. (internal quotations
and citation omitted).

Applying the first step of the applicable inquiry to Doc’s Privileges and Immunities
challenge, the Reinstatement Act clearly treats nonresident doctors of State B differently than
resident doctors. The reinstatement fee charged is 10 times more for nonresidents than it is for
residents. Moreover, the number of professional education courses is grossly disproportionate
between the two groups, greatly favoring the resident physicians. The court, however, will then
have to determine whether the discriminatory treatment of nonresident physicians implicates an
activity that is “fundamental to the promotion of interstate harmony.” Camden, 465 U.S. at 218.

“[T]he Privileges and Immunities Clause was intended to create a national economic
union.” Sup. Ct. of N.H. v. Piper, 470 U.S. 274, 279-80 (1985). Thus, the Court has repeatedly
held that the Privileges and Immunities Clause is implicated where there is “state discrimination
against nonresidents seeking to ply their trade, practice their occupation, or pursue a common
calling within the State.” Hicklin v. Orbeck, 437 U.S. 518, 524 (1978). Thus, Doc’s desire to
practice his occupation as a physician in State B certainly falls within the purview of a
fundamental right that implicates the Privileges and Immunities Clause. See Piper, 470 U.S. at

25
280-81 (holding that the practice of law is “important to the national economy,” and, therefore,
implicates the protection of the Privileges and Immunities clause); Toomer v. Witsell, 334 U.S.
385 (1948) (holding that South Carolina could not require nonresident fishermen to pay a license
fee of $2,500 for each shrimp boat owned when residents were charged only $25 per boat); Ward
v. Maryland, 79 U.S. 418 (1871) (invalidating a statute that required nonresidents to pay $300
per year for a license to trade in goods not manufactured in Maryland, while resident traders paid
a fee varying from $12 to $150).

Having established that Doc’s claim implicates a fundamental privilege, the question
turns to State B’s interest in treating resident and nonresident physicians differently. “Like many
other constitutional provisions, the privileges and immunities clause is not an absolute.” Toomer,
334 U.S. at 396. A law that implicates the Privileges and Immunities Clause may still be
constitutional “where (i) there is a substantial reason for the difference in treatment; and (ii) the
discrimination practiced against nonresidents bears a substantial relationship to the State’s
objective.” Piper, 470 U.S. at 284. In making this determination, the court will consider
whether less restrictive means are available to achieve the government’s objectives. Id. Finally,
“[a]s part of any justification offered for the discriminatory law, nonresidents must somehow be
shown to ‘constitute a peculiar source of evil at which the statute is aimed.’” Camden, 465 U.S.
at 222 (quoting Toomer, 334 U.S. at 398).

Here, the reason for the different treatment of resident and nonresident physicians is a
belief on the part of the legislature that resident physicians are more likely than nonresident
physicians to adhere to the highest standards of practice because they are practicing in their own
communities. However, the Supreme Court rejected a similar argument in Piper, where a
nonresident attorney challenged a rule limiting admission to the New Hampshire bar to residents
of the state. Piper, 470 U.S. at 276-77. In Piper, the state argued that resident attorneys were
more likely to “behave ethically” than are nonresident attorneys. Id. at 285. The Supreme Court
disagreed, noting that “[a] lawyer will be concerned with his reputation in any community where
he practices, regardless of where he may live. Furthermore, a nonresident lawyer may be
disciplined for unethical conduct.” Id. at 286. The Court ultimately concluded that the state in
Piper “neither advance[d] a ‘substantial reason’ for its discrimination against nonresident
applicants to the bar, nor demonstrate[d] that the discrimination practiced b[ore] a close
relationship to its proffered objectives.” Id. at 287.

In this case, State B’s proffered reason for subjecting nonresident physicians to higher
reinstatement fees than resident physicians is not a “substantial reason” for the discrimination.
As in Piper, it is likely a physician’s concern for patients’ well-being and his or her own
reputation would compel the physician to provide quality care regardless of residency.
Additionally, even if quality of care was determined to be a “substantial reason” sufficient to
justify the differing reinstatement fees, State B would not be able to show that the fees
themselves bear a “substantial relationship” to the State’s objective. As in Piper, a nonresident
physician is subject to the same external controls – such as malpractice claims and disciplinary
action – as are resident physicians. Finally, the facts presented state that there are proportionally
more malpractice claims and disciplinary actions brought against resident physicians than there
are against non-resident physicians. Thus, it would be unlikely that nonresident physicians

26
practicing in State B “constitute a peculiar source of the evil” of low quality of patient care. See
Camden, 465 U.S. at 22.

Because the Reinstatement Act discriminates against nonresident physicians in


connection with a fundamental privilege, and there is no substantial reason for the difference in
treatment between resident and nonresident physicians, it is likely that Doc will be successful in
challenging the Reinstatement Act as violating the Privileges and Immunities Clause.

2. The Court Will Likely Rule in Doc’s Favor and Conclude that the Severance
Agreement Does Not Contain an Enforceable Waiver of ADEA Claims

The Age Discrimination in Employment Act (ADEA) makes it unlawful for any private
employer with 20 or more employees to “. . . fail or refuse to hire or to discharge any individual or
otherwise discriminate against any individual with respect to his compensation, terms, conditions, or
privileges of employment, because of such individual’s age.” 29 U.S.C.A. §§ 623 (a)(1), 630(b) and
631. “In 1990, Congress amended the ADEA by passing the OWBPA [Older Workers Benefit
Protection Act]. The OWBPA provides: ‘An individual may not waive any right or claim under [the
ADEA] unless the waiver is knowing and voluntary. . . . [A] waiver may not be considered knowing
and voluntary unless at a minimum’ it satisfies certain enumerated requirements . . .” Oubre v.
Entergy Operations, Inc., 522 U.S. 422, 426 (1998) (citing 29 U.S.C. § 626(f)(1)).1 These
requirements are as follows:

(A) the waiver is part of an agreement between the individual and the employer
that is written in a manner calculated to be understood by such individual, or
by the average individual eligible to participate;

(B) the waiver specifically refers to rights or claims arising under this chapter;

(C) the individual does not waive rights or claims that may arise after the date the
waiver is executed;

(D) the individual waives rights or claims only in exchange for consideration in
addition to anything of value to which the individual already is entitled;

(E) the individual is advised in writing to consult with an attorney prior to


executing the agreement;
(F) (i) the individual is given a period of at least 21 days within which to
consider the agreement . . .;

1
While courts always required any enforceable waiver of ADEA claims to be “knowing and
voluntary,” prior to the enactment of the OWBPA, there was significant disagreement among the
federal appellate courts as to the applicable standard. Long v. Sears Roebuck & Co., 105 F.3d
1529, 1538 (3d Cir. 1997) (noting that historically the Fourth, Sixth and Eighth Circuits held that
state contract law principles controlled this issue, while the Third Circuit applied a more
stringent standard).

27
(G) the agreement provides that for a period of at least 7 days following the
execution of such agreement, the individual may revoke the agreement, and
the agreement shall not become effective or enforceable until the revocation
period has expired.

29 U.S.C. § 626(f).

Here, the Agreement that Doc executed fell woefully short of the statutory requirements
for an enforceable waiver of ADEA rights. Admittedly, the Agreement referenced waiver of
ADEA claims, seems to be written in a way that could be understood, and provided
consideration for the waiver in the form of a severance payment. However, the above facts are
silent as to whether the Agreement applied only to claims that had arisen prior to the date of the
agreement, and the facts also do not specifically indicate whether the Agreement advised Doc, in
writing, to consult with counsel. Absent these provisions, the waiver of Doc’s ADEA rights
would be unenforceable.

Most importantly, though, the statute requires that an individual be given at least 21 days
to consider any waiver of ADEA rights and that the agreement provide a 7 day revocation
period. Id. Here, Doc was told that he had to sign the Agreement before leaving work on the
same day that he was presented with the Agreement, thereby violating the 21 day consideration
requirement contained in the ADEA. Further, the Agreement states that it is “immediately and
irrevocably binding upon execution,” in violation of the 7 day revocation requirement.

Because the Agreement fails to meet the statutory requirements for a valid ADEA waiver,
the waiver is unenforceable, and HugeMed will not be able to successfully defend against Doc’s
lawsuit on this basis.

3. The Court Will Likely Deny HugeMed’s Motion for Summary Judgment Because
There Is a Genuine Issue of Material Fact as to Whether HugeMed’s Stated Reason
for Terminating Doc Is a Pretext for Discrimination.

Federal Rule of Civil Procedure 56 provides as follows:

(a) Motion for Summary Judgment or Partial Summary Judgment. A party


may move for summary judgment, identifying each claim or defense — or the
part of each claim or defense — on which summary judgment is sought. The
court shall grant summary judgment if the movant shows that there is no
genuine dispute as to any material fact and the movant is entitled to judgment
as a matter of law.

Regarding motions for summary judgment filed pursuant to Federal Rule 56, the
Supreme Court has explained:

[T[he plain language of Rule 56(c) mandates the entry of summary judgment,
after adequate time for discovery and upon motion, against a party who fails to

28
make a showing sufficient to establish the existence of an element essential to that
party's case, and on which that party will bear the burden of proof at trial. In such
a situation, there can be “no genuine issue as to any material fact,” since a
complete failure of proof concerning an essential element of the nonmoving
party's case necessarily renders all other facts immaterial. The moving party is
“entitled to a judgment as a matter of law” because the nonmoving party has
failed to make a sufficient showing on an essential element of her case with
respect to which she has the burden of proof.

Celotex Corp. v. Catrett, 477 U.S. 317, 322-23 (1986).

It is important, however, to keep in mind the role of juries in making determinations of


credibility, weighing evidence and drawing factual inferences. Anderson v. Liberty Lobby, Inc.,
477 U.S. 242, 255 (1986). “The evidence of the nonmovant is to be believed, and all justifiable
inferences are to be drawn in his favor.” Id. (citation omitted); see also Interstate Outdoor
Advertising, L.P. v. Zoning Bd. of Twp. of Mount Laurel, 706 F.3d 527, 530 (3rd Cir. 2013)
(stating that a court deciding a motion for summary judgment must “view the facts in the light
most favorable to the nonmoving party and draw all inferences in that party’s favor.”) (internal
quotation and citations omitted).

HugeMed has filed its motion for summary judgment on Doc’s ADEA claim. Thus, the court
must decide whether Doc, as the nonmoving party, “has failed to make a sufficient showing on an
essential element of [his] case . . . .” Celotex, 477 U.S. at 323. Again, the ADEA prohibits
discrimination against an employee “because of such individual’s age.” 29 U.S.C. § 623(a)(1). A
plaintiff seeking redress for age discrimination under the ADEA must prove that discrimination was
the “but-for” cause of the adverse employment action. Gross v. FBL Fin. Servs., Inc., 557 U.S. 167,
176 (2009), (“To establish a disparate-treatment claim under the plain language of the ADEA,
therefore, a plaintiff must prove that age was the ‘but-for’ cause of the employer's adverse
decision.”).

While the Supreme Court declined in Gross to expressly adopt the McDonnell Douglas
burden-shifting framework in ADEA cases, it also did not reject the application of this framework,
and the federal appellate courts have continued to apply it to cases where there is no direct evidence
of discrimination. See Sims v. MVM, Inc., 704 F.3d 1327, (11th Cir. 2013); Fleishman v. Cont'l Cas.
Co., 698 F.3d 598, 604 (7th Cir. 2012); Shelley v. Geren, 666 F.3d 599, 607 (9th Cir. 2012);
Gorzynski v. JetBlue Airways Corp., 596 F.3d 93, 106 (2d Cir. 2010); Jackson v. Cal-Western
Packaging Corp., 602 F.3d 374, 378 (5th Cir. 2010); Jones v. Okla. City Pub. Sch., 617 F.3d 1273,
1278 (10th Cir. 2010); Geiger v. Tower Auto., 579 F.3d 614, 622 (6th Cir. 2009); Smith v. City of
Allentown, 589 F.3d 684, 690-91 (3d Cir. 2009); Velez v. Thermo King de Puerto Rico, Inc., 585 F.3d
441, 446-47 (1st Cir. 2009); see also Gibson v. Am. Greetings Corp., 670 F.3d 844, 855 (8th Cir.
2012) (continuing to apply McDonnell Douglas to ADEA cases without discussion of Gross ), cert.
denied, 568 U.S. 885 (2012).

Here, Doc has admitted on the record that he has no direct evidence of age discrimination.
However, given the continued applicability by the courts of the McDonnell Douglas burden-shifting
analysis, Doc’s admission does not foreclose his ability to succeed on a claim under the ADEA.
Doc’s initial burden of production requires him to show: (1) that he is forty years old or older; (2)
that he suffered an adverse employment action; (3) that he was qualified for his position; and (4) that

29
he was replaced by another employee who was “sufficiently younger to support an inference of
discriminatory animus.” Smith, 589 F.3d at 689. Once Doc has satisfied the elements of a prima
facie claim of age discrimination, “the burden of production shifts to the employer to identify a
legitimate nondiscriminatory reason for the adverse employment action.” Id. at 690. Finally, the
burden of production will shift back to Doc to show that the legitimate nondiscriminatory reason
proffered by HugeMed is a pretext for age discrimination. Id. (citation omitted). “At all times,
however, the burden of persuasion rests with the plaintiff.” Id. (citation omitted).

There is evidence of record to satisfy Doc’s initial burden of establishing a prima facie claim
of discrimination. Doc is 66 years old, well above the age required by the ADEA to fall within its
protections. Second, Doc suffered an adverse employment action when his employment was
terminated by HugeMed. Third, Doc was qualified for his position – he was respected by colleagues,
and he was never subject to any malpractice or disciplinary actions. Finally, after he was terminated,
Doc was replaced by another physician 20 years his junior, which is certainly “sufficiently younger
to support an inference of discriminatory animus.”

HugeMed has also met its burden of production by proffering a legitimate, nondiscriminatory
reason for terminating Doc’s employment. HugeMed found an email sent from Doc’s HugeMed
email account inquiring about reinstatement of Doc’s medical license in a state where HugeMed does
not do business. From this, HugeMed assumed Doc was planning to leave HugeMed. HugeMed was
concerned that if Doc continued to work at HugeMed, he might try to convince patients to follow
Doc to a new practice.

Because HugeMed proffered a legitimate, nondiscriminatory reason for the termination of


Doc’s employment, Doc now must show that the proffered reason is pretext for discrimination. A
plaintiff may do so “by showing that the employer’s proffered explanation is unworthy of
credence.” Reeves v. Sanderson Plumbing Prods. Inc., 530 U.S. 133, 143 (2000) (quotation and
citation omitted). In Reeves, the employer cited “shoddy record keeping” as the reason for
terminating the plaintiff’s employment. Id. at 143. However, at trial, the employee made a
“substantial showing that [the employer’s] explanation was false,” and that the employee had
properly maintained the relevant records. Id. at 144-45. The plaintiff’s prima facie case of
discrimination coupled with the falsity of the employer’s stated reason for the plaintiff’s termination
was sufficient to support the jury verdict in favor of the plaintiff. Id. at 148.2

Here, there are several facts which, viewed in the light most favorable to Doc, would create
genuine issues of material fact regarding the falsity of HugeMed’s stated reason for terminating Doc.
First, there is no HugeMed office within 200 miles of State B. Thus, HugeMed’s stated concern that
Doc will lure patients away lacks credibility. Additionally, nine months lapsed between the time that
HugeMed reviewed the email that Doc sent to State B and the date that Doc’s employment was
terminated. If HugeMed was so concerned about Doc’s patient contacts, it would have made more
sense to terminate his employment as soon as HugeMed discovered the email inquiry.

2
In Reeves, the Supreme Court did leave open the possibility of “instances where, although the plaintiff has
established a prima facie case and set forth sufficient evidence to reject the defendant's explanation, no rational
factfinder could conclude that the action was discriminatory.” Id. This would most likely arise if there was
evidence that the employer proffered a false explanation for the discrimination in order to conceal a motive other
than discrimination. Id. However, there is no evidence of such concealment in the facts provided, and the general
rule of Reeves, as analyzed above, is applicable to the fact pattern.

30
It is likely that, viewing the evidence in the light most favorable to Doc, the court will
find there is enough evidence to create a genuine issue of material fact as to whether HugeMed’s
proffered reason for terminating Doc is a pretext for discrimination. Accordingly, the court will
likely deny HugeMed’s motion for summary judgment.

31
Question No. 4: Grading Guidelines

1. Constitutional Law

Comments: Applicants should demonstrate an understanding of the Privileges and Immunities


Clause and apply these principles to a set of facts and conclude that Doc’s challenge will likely
be successful.

6 points

2. Age Discrimination in Employment Act (ADEA)

Comments: Applicants should demonstrate an understanding of the requirements for an


enforceable waiver of ADEA claims pursuant to the Older Workers Benefit Protection Act’s
amendments to the ADEA and apply these principles to a set of facts and conclude that the court
would likely rule in Doc’s favor.

5 points

3. Federal Civil Procedure and ADEA

Comments: Applicants should demonstrate an understanding of the standard that federal courts
apply to motions for summary judgment under Federal Rule 56 and apply this standard to a set of
facts. Applicants should also demonstrate an understanding that the ADEA prohibits
discrimination on the basis of age, the causation requirements for ADEA claims, and the
application of the McDonnell Douglas burden-shifting framework to ADEA claims. Applicants
must apply these principles to a set of facts and conclude that Huge-Med’s motion would likely
fail.

9 points

32
Question No. 5

Bart was a world-famous professor at Big University (Big U) specializing in viniculture,

the science of producing wine-making grapes. To test his radical theories about rapidly growing

high-quality wine-making grapes on contaminated land, Bart orally agreed to buy Deadacre, a

large, well-defined tract of contaminated land in nearby Rural, Pennsylvania, owned by Sam.

Because Sam, a physician, was about to leave on an indefinite charity medical mission overseas,

Bart paid the purchase price in cash to Sam and they agreed to “sign the paperwork” upon Sam’s

return. Bart immediately moved from his home in Academia, Pennsylvania, and continuously

and exclusively resided at Deadacre. With the help of James, a Big U professor of enology, the

science of winemaking, Bart treated and cultivated the soil on Deadacre. Bart also made

numerous substantial permanent improvements to the property, including building a winery.

Within a short time, Bart had turned Deadacre into land of immeasurable value because he was

able to grow a type of grape that could be produced nowhere else that James made into a world-

class wine. Sam kept abreast of Bart’s actions at Deadacre via Internet news articles.

Bart and James opened a tasting room in the winery to market their newly created wine.

They entered into a written contract with EZ Construction (EZ) to build a wine cellar ancillary to

the tasting room at a cost of $200,000. The contract stated that EZ would install a cooling

system in the wine cellar using “Alpha” pipe which would be encased within the walls of the

cellar except where it had to be exposed. EZ inadvertently installed “Beta” pipe, a pipe of

virtually equal quality to Alpha pipe but having a slightly shorter life. When the mistake was

discovered, the wine cellar was nearly completed. Bart and James demanded that EZ rip out the

Beta pipe encased within the cellar’s walls and install Alpha pipe. EZ refused, citing the

prohibitive cost of removal and the near equal quality of the Beta pipe.

33
James hired Amy, a recent graduate of Big U’s wine science program, to be the wine

steward (sommelier) of the winery’s tasting room. Amy’s life-long dream was to become a

master sommelier, but she could not afford to travel and stay for a week at the expensive luxury

resort in Aspen, Colorado, where the rigorous master exam is administered. Knowing this,

James gratuitously promised to pay Amy’s travel, lodging, and other expenses related to the

exam. Amy, who was aware of the huge profits being earned by Bart and James from the tasting

room, relied upon James’ promise and incurred $15,000 of non-refundable travel, lodging, and

related expenses for her trip to Aspen to take the exam.

Bart decided to transfer his Academia home to Meg, his estranged daughter. Bart signed

and acknowledged a valid deed transferring title to Meg, and he prepaid the recording fee. Bart

then handed the deed to an agent of the title company stating, “Please record this deed for Meg

as soon as possible. It’s her house now. I hope that this makes up for all the time that I spent

with grapes instead of her.” Before the deed to the Academia home was recorded, Bart and Meg

had a terrible argument. Bart then called the agent and told him not to record the deed.

1. After returning from his overseas mission, Sam claimed that the oral agreement
for the sale of Deadacre was unenforceable and demanded that Bart immediately
leave the property. Bart sued Sam for specific performance. How would the
court likely rule?

2. Bart and James refused to pay EZ because Alpha pipe was not installed. EZ sued
for breach of contract demanding $200,000. Expert testimony during trial
conclusively demonstrated that Alpha pipe’s longer useful life made it worth
$5,000 more than Beta pipe. How should the court rule on the breach-of-contract
and damages claims?

3. Claiming a lack of funds, James reneged on his promise to pay Amy’s expenses
for her trip to Aspen to take the master sommelier exam. What theory of contract
law should Amy assert in a suit to enforce James’ promise to pay for her $15,000
in expenses and what is the likely result?

4. The agent told Meg about the unrecorded deed. Meg filed a proper action to
confirm that she had title to the Academia home. How should the court rule?

34
Question No. 5: Examiner’s Analysis

1. The court likely would rule that the oral agreement for the sale of Deadacre is
enforceable under the partial performance exception to the Statute of Frauds.

The Statute of Frauds requires that contracts for the sale of real property must be in
writing and signed by the granting party. 33 P.S. § 1; Rosen v. Rittenhouse Towers, 482 A.2d
1113, 1116 (Pa. Super. 1984). “The purpose of the Statute [of Frauds] is to prevent the
possibility of enforcing unfounded, fraudulent claims by requiring that contracts pertaining to
interests in real estate be supported by written evidence signed by the party creating the interest.”
Zuk v. Zuk, 55 A.3d 102, 107 (Pa. Super. 2012) (internal quotation and citations omitted).

“The Statute of Frauds is not a rule of evidence, but a declaration of public policy.”
Schuster v. Pa. Tpk. Comm’n, 149 A.2d 447, 451 (Pa. 1959) (citation omitted). “In the absence
of equities sufficient of themselves to take the case out of the statute, it operates as a limitation
upon judicial authority to afford a remedy unless renounced or waived by the party entitled to
claim its protection.” Kurland v. Stolker, 533 A.2d 1370, 1372 (Pa. 1987) (citation omitted).
Specifically, “the effect of the statute is to render oral contracts for the sale of real estate
unenforceable, although not invalid.” Hostetter v. Hoover, 547 A.2d 1247, 1250 (Pa. Super.
1988). The Statute of Frauds prevents a party seeking to enforce an oral contract for the sale of
real property from obtaining specific performance. Firetree, Ltd. v. Dep’t of Gen. Servs., 978
A.2d 1067, 1076 (Pa. Cmwlth. 2009) (citation omitted). In this case, Sam can oppose Bart’s
action for specific performance by contending that the oral agreement for the sale of Deadacre is
unenforceable and violates the Statute of Frauds because it is not in writing.

Even though the Statute of Frauds generally prevents the enforcement of contracts for the
sale of real property that are not in writing and signed by the party against whom enforcement is
sought, an oral agreement for the sale of real property can be enforced under the “partial
performance” exception to the Statute of Frauds. To demonstrate partial performance that would
take an oral agreement for the sale of real property out of the Statute of Frauds, the buyer must
provide full, complete and satisfactory proof of the terms of the agreement, including the amount
of consideration. Additionally, the buyer must show: (1) the boundaries and the quantity of the
land being purchased; (2) that possession of the property was taken in pursuance of the oral
contract; (3) at or immediately after the time that the contract was made that the change of
possession was notorious and has been exclusive, continuous and maintained; and (4) that there
was performance or part performance by the buyer which cannot be compensated in damages
and such as would make rescission of the oral contract inequitable and unjust. Kurland, 533
A.2d at 1373.

In this case, the facts state that Deadacre was a large, well-defined tract of land. The
facts also state that after paying the purchase price in cash to Sam, Bart immediately moved from
his home in Academia, Pennsylvania and continuously and exclusively resided at Deadacre.
Additionally, the facts state that Bart treated and cultivated the soil on Deadacre and made
numerous substantial permanent improvements to the property, including building a winery.
Finally, the facts state that Sam kept abreast of Bart’s actions at Deadacre via Internet news

35
articles. Based upon these facts, Bart should be able to satisfy the initial three requirements of
payment, defined boundaries and continuous, exclusive possession necessary for application of
the partial performance exception to the Statute of Frauds.

The final element needed to establish the partial performance exception requires that
valuable improvements were made that cannot be readily compensated in money damages and
are such that rescission of the oral agreement would be inequitable and unjust. Id. In Rader v.
Keiper, 132 A. 824 (Pa. 1926), the Pennsylvania Supreme Court discussed the type of
improvements that would take an oral contract for the sale of real property out of the Statute of
Frauds. In Rader, the Court stated, “Repairing and improving a dwelling house or outbuildings
are such improvements as any tenant for a term of years might make, and do not in any wise add
to the permanent value of the land.” 132 A. at 827 (adopting the lower court’s reasoning). In
Zuk v. Zuk, however, the Pennsylvania Superior Court affirmed a trial court’s determination that
the construction of the subbase on one thousand feet of access road, construction of a tool shed,
and construction of a cabin were “permanent and uncompensable” improvements sufficient to
make an oral agreement for the sale of land enforceable under the partial performance exception.
55 A.3d at 111.

In this case, the facts state that Bart made numerous, substantial, and permanent
improvements to the property, including building a winery. The improvements here are similar
in character and nature to those found sufficient under Pennsylvania case law to enforce an oral
agreement for the sale of real estate. But more significantly, the facts state that Bart’s treatment
and cultivation of the contaminated soil resulted in Deadacre becoming land of immeasurable
value because Bart was able to grow a type of grape on Deadacre that could be produced
nowhere else. In addition, the grapes grown on Deadacre were made into a world-class wine.
Given these facts, Bart has a strong argument that he has satisfied the final element of partial
performance exception because his part performance likely cannot be compensated in damages
and would make rescission of the oral contract with Sam for the sale of Deadacre inequitable and
unjust.

Although Sam can invoke the Statute of Frauds as a defense to Bart’s suit because the
agreement for the sale of Deadacre was not in writing, the court likely would rule that Bart’s oral
agreement with Sam for the sale of Deadacre is enforceable under the partial performance
exception to the Statute of Frauds.

2. The court should rule that Bart and James breached the contract by failing to pay
EZ. Bart and James were not discharged from their obligation to pay EZ because
EZ substantially performed the contract. EZ’s recovery of damages, however,
should be reduced by $5,000, the amount attributable to EZ’s failure to perform
exactly as required under the contract.

When performance of a duty under a contract is due, any unexcused failure to


perform that duty constitutes a breach of contract. Widmer Eng’g, Inc. v. Dufalla, 837 A.2d 459,
467 (Pa. Super. 2003), appeal denied 852 A.2d 313 (Pa. 2004); see also RESTATEMENT
(SECOND) OF CONTRACTS, § 235 (2), cmt. b (Am. Law Inst. 1981).

36
If a breach [of contract] constitutes a material failure of performance, the non-
breaching party is relieved from any obligation to perform; thus, a party who has
materially breached a contract may not insist upon performance of the contract by
the non-breaching party. Conversely, a party might breach the contract but still
substantially perform its obligations under the agreement. In that case, the breach
is deemed nonmaterial and the contract remains in effect. The breaching party
retains the right to enforce the contract and demand performance; the non-
breaching party has no right to suspend performance.

McCausland v. Wagner, 78 A.3d 1093, 1101 (Pa. Super. 2013) (citations omitted).

“The doctrine of material breach is simply the converse of the doctrine of substantial
performance. Substantial performance is performance without a material breach, and a material
breach results in performance that is not substantial.” Gen. Motors Corp. v. New A.C. Chevrolet,
Inc., 263 F.3d 296, 317 n. 8 (3d Cir.2001), quoting E.A. FARNSWORTH, FARNSWORTH ON
CONTRACTS, § 8.16 (1990). The doctrine of substantial performance is “intended for the
protection and relief of those who have faithfully and honestly endeavored to perform their
contracts in all material and substantial particulars, so that their right to compensation may not be
forfeited by reason of mere technical, inadvertent or unimportant omissions or defects.” First
Mortg. Co. of Pa. v. Carter, 452 A.2d 835, 837 (Pa. Super. 1982). Thus, the duty of Bart and
James to pay EZ can be discharged only if EZ’s failure to construct the wine cellar with Alpha
pipe as specified in the contract constitutes a material breach. If EZ has substantially performed
the contract and is not in material breach, then the duty of Bart and James to perform is not
ended and they will be required to pay the $200,000 contract price, less the amount caused by
EZ’s failure to perform exactly as required under the contract. See, JOHN E. MURRAY, JR.
MURRAY ON CONTRACTS, § 108 [B] [2] (5th ed. 2011).

In determining whether a contractual breach is material, Pennsylvania courts look to the


factors set forth in Section 241 of the Restatement (Second) of Contracts. Int’l Diamond Imps.,
Ltd. v. Singularity Clark, L.P., 40 A.3d 1261, 1271 (Pa. Super. 2012). The Restatement factors
include the following:

(a) the extent to which the injured party will be deprived of the benefit which he
reasonably expected;
(b) the extent to which the injured party can be adequately compensated for the
part of that benefit of which he will be deprived;
(c) the extent to which the party failing to perform or to offer to perform will
suffer forfeiture;
(d) the likelihood that the party failing to perform or to offer to perform will cure
his failure, taking account of all the circumstances including any reasonable
assurances;
(e) the extent to which the behavior of the party failing to perform or to offer to
perform comports with standards of good faith and fair dealing.

RESTATEMENT (SECOND) OF CONTRACTS, § 241 (Am. Law Inst. 1981). Whether a breach of
contract is material is generally a question of fact for a jury to decide. Int’l Diamond Imps., Ltd.,

37
40 A.3d at 1272.

The facts here are virtually identical to those in Jacob & Youngs, Inc. v. Kent, 129 N.E.
889 (N.Y. Ct. App. 1921), the landmark case dealing with the doctrine of substantial
performance. In Jacob & Youngs, a contract for the building of a house specified that all of the
wrought-iron pipe used in the structure would be of “Reading manufacture.” By error, pipe of
“Cohoes manufacture,” which was identical in quality to the desired pipe, was installed. The
mistake of using Cohoes pipe was not discovered until after the house was completed and
replacement of the pipe would have required demolition of large portions of the completed
house. The contractor refused to replace the pipe and eventually sued to recover his final
progress payment withheld by the owners. In finding for the contractor, the court recognized
that, while a party to a contract generally has a duty of full performance, a failure to fully
perform, “both trivial and innocent, will sometimes be atoned for by allowance of the resulting
damage, and will not always be the breach of a condition to be followed by a forfeiture.” Jacob
& Youngs, 129 N.E. at 890. The court then suggested criteria for determining whether a breach
was material or performance substantial stating, “We must weigh the purpose to be served, the
desire to be gratified, the excuse for deviation from the letter, the cruelty of enforced adherence.”
Id. at 891. The court concluded, “The rule that gives a remedy in cases of substantial
performance with compensation for defects of trivial or inappreciable importance has been
developed by the courts as an instrument of justice. The measure of the allowance must be
shaped to the same end.” Id. at 892. Based upon Jacob & Youngs, a court would rule that EZ
substantially performed the contract, but that EZ’s recovery should be reduced by the value
attributable to its use of Beta as opposed to Alpha pipe.

Moreover, application of the modern Restatement factors for determining the materiality
of a breach to the facts here likely would produce the same result as in Jacob & Youngs. In this
case, there are no facts which suggest that Bart and James were truly deprived of a reasonably
expected benefit because the wine cellar was constructed with Beta pipe instead of Alpha pipe as
promised in the contract. To the contrary, the facts state that the Beta pipe was virtually of equal
quality to the Alpha pipe. Additionally, Bart and James can be adequately compensated for any
lost benefit of not having Alpha pipe in the wine cellar by having the diminished value of the
Beta pipe deducted from the contract price. Finally, if the duty of Bart and James to perform was
excused, EZ would suffer a forfeiture of the entire $200,000 contract price, an amount clearly
disproportionate to any loss suffered by Bart and James.

Applying either the standards in the Jacob & Youngs case or the Restatement factors for
determining the materiality of a breach to the stated facts, the court in this case would likely
conclude that EZ’s failure to use Alpha pipe as specified was not a material breach of contract
and that EZ had substantially performed. Therefore, Bart and James were not relieved of their
duty to pay EZ. EZ’s counterclaim for the $200,000 contract price, however, would be reduced
by $5,000, the amount of the damages suffered by Bart and James due to EZ’s partial breach of
the contract for failure to construct the wine cellar with Alpha pipe.

3. Amy can successfully assert the doctrine of promissory estoppel to enforce James’
gratuitous promise to pay for her travel expenses.

38
The issue initially presented under the facts is whether an oral contract was created
between James and Amy. “In order for a contract to be formed, there must be an offer,
acceptance, and an exchange of consideration.” Pa. Workers' Comp. Judges Prof’l Ass'n v. Exec.
Bd., 39 A.3d 486, 493 (Pa. Cmwlth. 2012). Consideration requires a finding of legal value and
bargained-for-exchange. JOHN E. MURRAY, JR. MURRAY ON CONTRACTS, § 56 (5th ed. 2011).
In this case, the facts state that James’ promise to pay Amy’s expenses to take the master
sommelier exam in Aspen was gratuitous. Therefore, there is no consideration to have an
enforceable contract because Amy’s detriment did not induce James’ promise. See Pennsy
Supply, Inc. v. American Ash Recycling Corp., 895 A.2d 595, 601 (Pa. Super. 2006).

As an oral contract does not exist because of a lack of consideration, Amy instead can
seek to enforce James’ promise by bringing an action based upon the doctrine of promissory
estoppel. Promissory estoppel is an outgrowth of equitable estoppel, but, unlike equitable
estoppel, it may serve as an independent cause of action. Paul v. Lankenau Hosp., 543 A.2d
1148, 1152 (Pa. Super. 1988), judgment aff’d in part and rev’d in part, 569 A.2d 346 (Pa. 1990).
“Detrimental reliance is another name for promissory estoppel.” Travers v. Cameron Cty. Sch.
Dist., 544 A.2d 547, 550 (Pa. Cmwlth. 1988). The doctrine of promissory estoppel allows a
party under certain circumstances to enforce a promise even though that promise is not supported
by consideration. Crouse v. Cyclops Indus., 745 A.2d 606, 610 (Pa. 2000). Pennsylvania has
long recognized promissory estoppel as a vehicle by which a promise may be enforced in order
to remedy an injustice. See Fried v. Fisher, 196 A. 39 (Pa. 1938).

Pennsylvania has adopted the statement of the doctrine of promissory estoppel as it


appears in Section 90 of the Restatement (Second) of Contracts. See Murphy v. Burke, 311 A.2d
904, 908 (Pa. 1973). Thus, to establish a cause of action based upon promissory estoppel, a
plaintiff must prove three elements: (1) the promisor made a promise that would reasonably be
expected to induce action or forbearance on the part of the promisee; (2) the promisee actually
took action or refrained from taking action in reliance on the promise; and (3) injustice can be
avoided only by enforcing the promise. Holewinski v. Children’s Hosp. of Pittsburgh, 649 A.2d
712, 714 (Pa. Super. 1994), appeal denied 659 A.2d 560 (Pa. 1995); RESTATEMENT (SECOND)
OF CONTRACTS, § 90 (1). “One of the factors that a court may consider in determining whether
a promisee has satisfied this element is ‘the reasonableness of the promisee’s reliance.’”
Shoemaker v. Cmwlth. Bank, 700 A.2d 1003, 1008 (Pa. Super. 1997), citing Thatcher’s Drug
Store of West Goshen, Inc. v. Consolidated Supermarkets, Inc., 636 A.2d 156, 160 (Pa. 1994),
quoting RESTATEMENT (SECOND) OF CONTRACTS, § 90 cmt. b.

The facts state James made his gratuitous promise knowing that Amy’s life-long dream
was to become a master sommelier and that she lacked the necessary funds to travel to take the
exam. The facts further state that Amy was aware of the huge profits being earned by Bart and
James from the operation of the tasting room. Given these facts, the first element for a cause of
action based upon promissory estoppel would be satisfied because James’ promise to pay Amy’s
travel, lodging, and other expenses related to the exam is one that would reasonably be expected
to induce action on Amy’s part. Additionally, the second element for a promissory estoppel
claim would be satisfied because the facts state that Amy actually took action in reliance on
James’ promise by incurring $15,000 of non-refundable travel, lodging, and related expenses to
travel to Aspen, Colorado to take the exam. Finally, the facts support a finding that enforcing

39
James’ promise would prevent an injustice. Because James possessed the financial wherewithal
to make good on his promise to pay for Amy’s travel expenses without any hardship to him, the
interest of justice would militate in favor of enforcing his promise.

In short, despite the absence of an enforceable oral contract, it is likely that Amy can
successfully use the doctrine of promissory estoppel to enforce James’ promise to pay for her
travel expenses.

4. The court should rule that Meg has title to the Academia home because the deed
was constructively delivered through a third party.

For an inter vivos gift of realty to be valid, “there must coincide: (1) a donative intent
upon the part of the grantor, i.e., an intent to make a gift to the grantee then and there, when the
deed was executed; [and] (2) a delivery of the deed to the grantee.” Loutzenhiser v. Doddo, 260
A.2d 745, 747 (Pa. 1970). (citations omitted).

“[D]elivery of a deed is necessary in order to render it legally operative.” Herr v. Bard,


50 A.2d 280, 281 (Pa.1947). “Whether there has been a delivery of a deed is a question of fact . .
. [and] depends upon the intention of the grantor as shown by his words and actions and by the
circumstances surrounding the transaction.” Abraham v. Mihalich, 479 A.2d 601, 602 (Pa.
Super. 1984).

To affect a delivery, it is not necessary for a grantor actually or manually to give the deed
to the grantee. Chambley v. Rumbaugh, 5 A.2d 171, 173 (Pa. 1939). A constructive delivery may
be found where the grantor evidences an intent to divest himself of all dominion and control over
the property and invest the grantee therewith. Fiore v. Fiore, 174 A.2d 858, 859 (Pa. 1961).

One circumstance under which a valid delivery may occur is when the grantor places the
deed in the possession of a third party. In such a case, the legal delivery date relates back to the
time of the initial delivery to the third party if that is necessary to effectuate the grantor’s
intention. In re Rynier’s Estate, 32 A.2d 736, 738 (Pa. 1943). But the mere handing of the
executed deed to a third party who is the agent of the grantee, without more, does not constitute
valid delivery. As the Pennsylvania Supreme Court has observed, “In order for the delivery to
be effectual and to result in a culmination of the transition of the title, there must be an express
and definite instruction that the deed is to be given to the grantee then or at some future time.”
Fiore, 174 A.2d at 860 (citations omitted). “[I]f . . . such instructions were given, or if the
attendant facts and circumstances indicate that the intent of the grantor was that delivery should
be made by the third [party] to the grantee, and if the grantor parted with control, [the] delivery is
. . . valid [even though] actual enjoyment of the property may be postponed.” Chambley, 5 A.2d
at 173.

Here, the facts state that Bart signed and acknowledged a valid deed transferring title to
Meg and prepaid the recording fee. Bart then handed the deed to an agent of the title company
stating, “Please record this deed for Meg as soon as possible. It’s her house now. I hope that this
makes up for all the time that I spent with grapes instead of her.” In Mower v. Mower, 80 A.2d
856, 858 (Pa. 1951), the Pennsylvania Supreme Court concluded that a grantor had parted with

40
all control and had delivered a deed when he executed and acknowledged the instrument, paid
the recording fee, and left the deed with a third party with instructions to have it recorded.
Similar to Mower, the facts in this case support the conclusion that Bart intended to divest
himself of all dominion and control over his Academia home and transfer title to Meg when he
handed the deed to the agent for recording.

Finally, the failure to record the deed and Meg’s lack of prior knowledge about the
transaction does not affect delivery. “The recording of a deed is not essential to its validity or
the transition of title.” In re Cray Estate, 44 A.2d 286, 287 (Pa.1945). Since the grantee always
can ratify the original delivery by subsequent acceptance, the grantee need not have knowledge
of the transaction. Fiore, 174 A.2d at 859.

Based upon his words and actions, a court probably would conclude that Bart intended to
give up dominion and control over his Academia home and to transfer title to Meg and that a
valid delivery of the deed occurred through a third party. Therefore, the court should rule that
Meg has title because Bart made a valid inter vivos gift of the property.

41
Question No. 6

T Township in C County, Pennsylvania, consists mainly of large tracts of wooded land dotted with a

few weekend cabins. Its unspoiled beauty and quiet solitude make it an attractive getaway area. Neither T

Township nor C County has any ordinances regulating land use, noise, odors or trash burning.

Peter and David own adjoining tracts of land in T Township. Peter has a small cabin on his property

that he and his family use every weekend and on holidays and vacations. David has a large open pavilion

on his property. Originally, he used it only for occasional family events, and the two neighbors coexisted

peacefully. In recent years, however, David began hosting large parties at his pavilion for his all-terrain-

vehicle (ATV) club. The parties occurred mainly on weekends and holidays, and therefore largely

coincided with Peter’s visits to his cabin.

The parties at David’s pavilion started during the day and often lasted late into the night. Loud

music, noise from motorcycles and ATVs, and shouted profanities made it impossible for Peter and his

young children to spend time enjoying the outdoors. Because David’s pavilion was very close to Peter’s

property line and distant from any other occupied areas, the noise only affected Peter and his family. The

noise was frequently disruptive even inside Peter’s cabin. He and his family were forced to keep their

windows and doors closed, even on the hottest summer nights, in order to get to sleep. The resulting heat

and stuffiness made it necessary to install air conditioning in the cabin, which greatly increased the electric

bills for the cabin and also reduced the woodsy ambiance of sleeping with the windows open. Peter asked

David to modify the noise levels, profanity, and ending times for parties. David refused.

Relations between the parties deteriorated over the next couple of years. Knowing that David’s

property is downhill and downwind from his, Peter cleared a narrow strip of land along the length of his

shared property line with David’s land and gave permission for a farmer from a neighboring area to spread

manure there. The odor of the manure drifted down to David’s pavilion.

42
(1) Assume that Peter properly filed a civil complaint against David in the C County Court of Common
Pleas based on all the noise issues that had arisen over time.

(a) Other than trespass and negligence, what tort claim is most closely supported by the facts that
Peter can assert against David based on the noise and with what likely result?
(b) If Peter is successful on his above claim, and he seeks a permanent injunction to restore
quiet, would the court grant the injunction?

(2) Assume that Peter’s initial manure spreading activities were legal and not tortious. Assume also that
later, a heavy storm washed much of the manure onto David’s land. The manure’s physical presence
on David’s property caused no pollution or irreparable damage. However, there was a sufficient
quantity present to interfere with traction for motorcycles and ATVs traversing that part of David’s
property. David paid a cleanup service that effectively removed the manure.

(a) What intentional tort can David assert against Peter in a counterclaim based on the physical
intrusion of the manure onto his land?
(b) Assuming David is successful on his counterclaim, what damages would a court likely
award?

(3) Assume that the case has proceeded through the pretrial process, that the pleadings and discovery are
closed, and that a trial date has been set. In his pretrial statement filed with the court shortly before
trial, David states for the first time that he intends to defend against Peter’s claim on the basis that
Peter failed to bring his action within the time specified by the applicable statute of limitations. If
Peter files a motion in limine to preclude that defense, what basis should he assert under the rules of
civil procedure, and how should the court rule?

(4) As part of his defense against David’s counterclaim, Peter intends to argue that the encroachment of
the manure onto David’s land was inadvertent and unforeseeable. To rebut that argument, David
wants to introduce testimony from the farmer about a discussion with Peter in which the farmer
claims Peter stated that it was only a matter of time before the manure would wash onto David’s
property. Assume that this proposed testimony is relevant and that Peter raises a hearsay objection.

(a) Is the farmer’s statement hearsay if it is offered to show that Peter intended the manure
would be washed onto David’s land?
(b) Assume, for this sub-question only, that the court finds the statement is hearsay and is not
admissible as a present sense impression or statement of intent. Applying only Pennsylvania
hearsay exceptions, is there another basis to admit the farmer’s proposed testimony?

43
Question No. 6: Examiner’s Analysis

1.(a) Peter likely can prevail in a claim of private nuisance against David based on the extreme
noise.

A nuisance arises from unreasonable, unwarrantable, or unlawful use of one’s own property that
damages another’s right to the use of the other’s property, producing material annoyance, inconvenience, or
discomfort, and consequential damages. Kramer v. Pittsburgh Coal Co., 341 Pa. 379, 380-81, 19 A.2d 362,
363 (1941). Private nuisance generally does not involve a physical trespass. See Golen v. Union Corp., 718
A.2d 298, 300 (Pa. Super. 1998).

The tort of private nuisance is governed in Pennsylvania by section 822 of the Restatement (Second)
of Torts:

One is subject to liability for a private nuisance if, but only if, his conduct is a legal
cause of an invasion of another’s interest in the private use and enjoyment of land, and the
invasion is either

(a) intentional and unreasonable, or

(b) unintentional and otherwise actionable under the rules controlling liability for
negligent or reckless conduct, or for abnormally dangerous conditions or
activities.

Restatement (Second) of Torts § 822 (Am. Law Inst.1979). See Kembel v. Schlegel, 478 A.2d 11, 14, n.3
(Pa. Super. 1984) (adopting section 822 as Pennsylvania law).1 Thus, under the Restatement, either
intentional or reckless conduct will support a private nuisance claim.2

An invasion of another’s interest in the use and enjoyment of land is intentional if the actor either
acts with the purpose of causing the invasion or knows it is resulting or substantially certain to result from
his conduct. Youst v. Keck’s Food Serv., Inc., 94 A.3d 1057, 1072-73 (Pa. Super. 2014) (quoting
Restatement (Second) of Torts § 825). An invasion is unreasonable if the gravity of the harm it causes
outweighs any utility of the actor’s conduct, or if the harm is “serious and the financial burden of
compensating for this and similar harm to others would not make the continuation of the conduct not
feasible.” Youst, 94 A.3d at 1073 (quoting Restatement (Second) of Torts § 826).

In addition, section 821F of the Restatement requires “significant harm” to support a private
nuisance claim. Restatement (Second) of Torts § 821F (1979); see Kembel, 478 A.2d at 15. Significant
harm is harm of importance, not slight or petty inconvenience. There must be real and appreciable

1
The Pennsylvania Supreme Court previously adopted the prior version of section 822. See Waschak v. Moffat, 379 Pa. 441, 454,
109 A.2d 310, 317 (1954). However, the differences between the two versions are minimal.
2
Although the Restatement also allows imposition of liability based on negligence or abnormally dangerous activity, neither is
supported under the facts given for this question. There is no indication of other than intentional conduct, and none of the parties’
potentially tortious conduct implicated any dangerous activity.

44
interference with the plaintiff’s use of his own land. Restatement (Second) of Torts § 821F cmt. c. Whether
the harm is significant is determined based on the opinion of a “normal person” living in the particular
locality. Restatement (Second) of Torts § 821F cmt. d. “If normal persons living in the community would
regard the invasion in question as definitely offensive, seriously annoying or intolerable, then the invasion is
significant.” Id. Noise that is inconsistent with a quiet area and keeps neighbors from sleeping can be
sufficiently significant to support a nuisance claim. See Anderson v. Guerrein Sky-Way Amusement Co.,
346 Pa. 80, 82-84, 29 A.2d 682, 684 (1943) (discussing excessive noise in a quiet residential area).

Applying these principles to the question, David’s conduct supports a private nuisance claim by
Peter. Regarding the noise generated by the partiers, it was loud enough and frequent enough to cause
significant disruption of comfort, loss of use and enjoyment of outdoor space, and loss of sleep. David was
aware of the noise and that it was invading Peter’s interest in the use and enjoyment of his land; however,
David refused to take any reasonable steps to direct his guests to reduce the volume or duration of the noise,
making the ongoing harm actionable in nuisance, whether as intentional or reckless conduct. The party
noise would constitute significant harm by the standards of normal persons living in T Township, who are
accustomed to the quiet of rural woodlands. A jury would likely find the harm to Peter and his family
outweighed the utility of allowing David’s guests to hold parties at David’s pavilion.

1.(b) A court is likely to award injunctive relief.

An injury to property that is not permanent in character is a temporary nuisance. See Milan v.
Bethlehem, 94 A.2d 774, 776 (Pa. 1953).

A temporary private nuisance is an appropriate case for an award of injunctive relief to control or
stop the offending use of the defendant’s property. See McKees Rocks v. Rennekamp Supply Co., 344 Pa.
443, 445, 25 A.2d 710, 711 (1942). A court may enjoin a nuisance where it cannot be adequately
compensated by damages, or where by its nature it will create a recurring grievance that will lead to a
multiplicity of suits. See Mowday v. Moore, 133 Pa. 598, 611, 19 A. 626 (1890). A use of a defendant’s
property creating disturbing noise in a residential neighborhood is subject to a permanent injunction.
McKees Rocks, 25 A.2d at 711.

To establish a right to a permanent injunction (as opposed to a preliminary injunction), the plaintiff
must show a clear right to relief, but need not show either irreparable harm or a need for immediate relief.
Youst, 94 A.3d at 1078 (quoting J.C. Erlich Co. v. Martin, 979 A.2d 862, 864 (Pa. Super. 2009)). The court
may issue a permanent injunction if it is necessary to prevent a legal wrong for which there is no adequate
remedy at law. Id. In doing so, the court will balance the parties’ competing interests in using their
respective properties. See 1621, Inc. v. Wilson, 402 Pa. 94, 108, 166 A.2d 271, 278 (1960). The harm
suffered by the plaintiff must be substantial and repeated. See Wellspan Health v. Bayliss, 869 A.2d 990,
995 (Pa. Super. 2005). The court must also consider whether an injunction serves the public interest. See
Rollins Outdoor Advertising, Inc. v. Zoning Bd. of Adjustment, 108 Pa. Commw. 277, 283, 529 A.2d 99, 102
(1987).

Applying these rules to the facts of the question, Peter may obtain injunctive relief to stop or reduce
the noise because the nature of the nuisance is one that can be abated by confining the noise to reasonable
times and levels. He is clearly entitled to relief from the extreme noise; balancing the parties’ interests,
Peter’s interest in family time and an appropriate atmosphere for his children outweighs David’s interest in

45
allowing his friends to have noisy parties. The noise is temporary in character, making it amenable to
abatement by David. A court could reasonably conclude that money damages would be inadequate to
compensate Peter for the loss of use of the property, especially given the unique character of real property in
general and this pristine rural area in particular. Finally, the public interest is served by supporting the right
of real property owners to the peaceful enjoyment of their properties. See id.

The facts of this case are consistent with those of the decisions cited above in which courts have
permanently enjoined private nuisances. The court has discretion to tailor its injunctive relief to the
minimum relief necessary to protect Peter’s interests.

2.(a) David may assert a counterclaim for trespass based on the incursion of manure onto his
land.
One is subject to liability to another for trespass, irrespective of whether he thereby causes
harm to any legally protected interest of the other, if he intentionally

(a) enters land in the possession of the other, or causes a thing or a third person
to do so . . . .

Gavin v. Loeffelbein, 161 A.3d 340, 355 (Pa. Super. 2017) (quoting Restatement (Second) of Torts §
158 (Am. Law Inst. 1965)). As described in the Restatement, the tort has two elements: (1) the
“entry element,” actual interference with another’s right of exclusive possession; and (2) intent.
Possession may be actual or constructive. Griffin v. Delaware & Hudson Co., 257 Pa. 432, 101 A.
750 (1917). The requisite intent is present when the actor enters or causes entry on the land,
whether or not he knows he is not entitled to do so. See Kopka v. Bell Telephone Co., 371 Pa. 444,
450, 91 A.2d 232, 235 (1952). “Intent” does not require knowledge that one is trespassing, but
simply intent to enter or cause entry to land. Id. Actual harm resulting from the entry is not a
required element of the tort of trespass. Liberty Place Retail Assocs., L.P. v. Israelite Sch. of Univ.
Practical Knowledge, 102 A.3d 501, 506 (Pa. Super. 2014) (quoting Restatement (Second) of Torts
§ 158.

A trespass action will lie for an invasion of another’s land, such as dumping of soil or
causing runoff. See Cassel-Hess v. Hoffer, 44 A.3d 80, 87 (Pa. Super. 2012) (discussing and
contrasting Beach St. Corp. v. A.P. Constr. Co., 658 A.2d 379, 379 (Pa. Super. 1995) and Graybill
v. Providence Twp., 593 A.2d 1314, 1316-17 (Pa. Cmwlth. 1991), aff’d per curiam, 533 Pa. 61, 618
A.2d 392 (1993)).

Applying the rule to this question, Peter’s conduct may support a trespass claim by David.
Peter directed the placement of manure along the property line. If he did so with the substantial
certainty that running water from a heavy storm would carry the manure downhill onto David’s
property, then David could show that Peter intended the manure to trespass onto David’s property.
The entry element would also be met, as Peter would have caused the manure to enter David’s land.
Under those circumstances, David would likely be able to show Peter trespassed on David’s land.

46
2.(b) The court will likely award damages to David for the cost of repair of the damaged
land.

Where the damage to property from a trespass is repairable, the correct measure of damages
is the cost of repair. See Rabe v. Shoenberger Coal Co., 213 Pa. 252, 255-56, 62 A. 854, 855 (1906).
Under the facts of the question, damages should be the cost of removing the manure, which David hired a
cleanup service to do. The facts as given do not indicate any other damage to David’s property. Therefore,
David’s damages would be limited to the cost of removing the manure.

3. Peter should assert that David waived the affirmative defense of the statute of limitations by
failing to plead it, and the court will likely rule in Peter’s favor.

Pennsylvania Rule of Civil Procedure 1030 provides, in pertinent part: “[A]ll affirmative defenses
including . . . statute of limitations . . . shall be pleaded in a responsive pleading under the heading ‘New
Matter’. . . .” Pa. R.C.P. No. 1030(a) (2017). Rule 1032 provides, in pertinent part: “A party waives all
defenses and objections which are not presented either by preliminary objection, answer or reply . . . .” Pa.
R.C.P. No. 1032(a) (2017). Thus, a defendant who fails to raise the statute of limitations in new matter
waives that affirmative defense. See Solomon v. A. Julian, Inc., 450 A.2d 130, 132 (Pa. Super. 1982).

Applying this rule to the facts of the question, David did not plead the statute of limitations as a
defense in new matter as required by Pa. R.C.P. No. 1030 or otherwise raise it during the pleading stage of
the action. In fact, he waited until just before trial to raise the defense, mentioning it for the first time in his
pretrial statement. In this circumstance, the court should find the defense was waived and should grant
Peter’s motion in limine and preclude David’s assertion of an affirmative defense based on the statute of
limitations. See Solomon, 450 A.2d at 132 (trial court properly refused to allow amendment to new matter
to assert statute of limitations defense three years after filing of complaint, when trial was imminent).

4.(a) The statement is not hearsay because it is not offered for its truth.

“‘Hearsay’ means a statement that (1) the declarant does not make while testifying at the current trial
or hearing; and (2) a party offers in evidence to prove the truth of the matter asserted in the statement.” Pa.
R.E. 801(c) (2017). Pa. R.E. 802 states the general rule that hearsay is not admissible except as provided in
other evidence rules.

As the definition in Rule 801(c) suggests, an out-of-court statement not offered for its truth is not
hearsay. Castellani v. Scranton Times, 124 A.3d 1229, 1244 (Pa. 2015) (citing Commonwealth v. Puksar,
559 Pa. 358, 368, 740 A.2d 219, 225 (1999)). When an extrajudicial statement is offered for a purpose
other than proving the truth of its contents, it is not hearsay and is not excludable under the hearsay rule.
Thus, statement are admissible to establish ill-will or motive where they are not being offered for the truth
of the matter contained therein. Puksar, 559 Pa at 368, 740 A.2d at 225. The Castellani court stated, an
out-of-court statement offered simply to show notice to the declarant is not hearsay. Castellani, 124 A.3d at
1244 (additional citations omitted).

47
Applying this rule to the facts of the question, the farmer’s statement would be offered not to prove
that the manure would inevitably be washed onto David’s land, but only to show Peter’s motive and that he
had notice of the likelihood that it could do so. Therefore, the farmer’s proposed testimony is not hearsay.

4.(b) The party statement exception to the hearsay rule would apply in this case.

Rule 803 provides a long list of exceptions to the hearsay rule. Rule 803(25) provides a hearsay
exception for party admissions. “An Opposing Party’s Statement. The statement is offered against an
opposing party and . . . was made by the party in an individual or representative capacity . . . .” Pa.R.E.
803(25) (2017). See also Alessandro v. Workers’ Comp. Appeal Bd. (Precision Metal Crafters, LLC), 972
A.2d 1245, 1252 (Pa. Cmwlth. 2009) (an admission of an opposing party can be used as substantive
evidence).

Applying this exception to the facts and assumptions of the question, the farmer’s proposed
testimony would be offered to show that Peter intended or could reasonably foresee that the manure on his
land would run off onto David’s. As a statement allegedly made directly by Peter to the farmer, the prior
statement fits this hearsay exception.3

3
Although Rule 803.1 provides a hearsay exception for prior inconsistent statements of a declarant-witness, that rule applies only
if the prior statement was made under oath, in a signed writing, or in a verbatim contemporaneous recording. See Pa. R.E.
803.1(1). Under the facts of this question, there is no indication that Peter’s alleged prior statement fits any of those three
categories. Therefore, Rule 803.1 does not provide an applicable hearsay exception.

Rule 613 also authorizes use of a prior inconsistent statement to impeach a witness’s credibility, but this is not a hearsay
exception and expressly does not apply to statements defined in Rule 803(25). See Pa. R.E. 613(a), (b). Therefore, this rule is
also inapplicable to this question.

48
Question No. 6: Grading Guidelines

1. Torts – Private Nuisance

Comments: (a) The candidate should recognize that the appropriate claim by Peter against David is
private nuisance. The candidate should state the elements of private nuisance and conclude that David’s
conduct created a private nuisance.
(b) The candidate should recognize the requirements for injunctive relief and apply those
requirements to the facts to reach a well-reasoned conclusion.

6 Points

2. Counterclaim – Trespass

Comments: (a) The candidate should recognize that a claim of trespass may lie for the runoff of manure
onto David’s land. The candidate should recognize the elements of trespass and apply those elements to the
facts to reach a well-reasoned conclusion.
(b) The candidate should recognize the correct measure of damages would likely be the cost
of repair.

6 Points

3. Civil Procedure – Affirmative Defense Not Pleaded Is Waived

Comments: The candidate should recognize that the statute of limitations is an affirmative defense and that
raising it first in a pretrial statement constitutes waiver.

3 Points

4. Evidence – Exceptions to the Hearsay Rule

Comments: (a) The candidate should recognize that the statement is not hearsay.
(b) The candidate should recognize that the party statement exception to the hearsay rule is
applicable.

5 Points

49
PT
Question Number 3
PLACE BAR CODED APPLICANT LABEL HERE

on Examplify

Supreme Court of Pennsylvania


Pennsylvania Board of Law Examiners

Pennsylvania Bar Examination


February 24
February 27 and
and 28,
25,2018
2015

PERFORMANCE TEST
February
February 27,
24, 2018
2015

Use GRAY covered book for your answer to the Performance Test.
2018 Pa
© 2015 Board Of Law Examiners
Pennsylvania Board of Law Examiners

50
Table of Contents

FILE

1. Assignment Memorandum .........................................................................................................1

2. Typed Notes from Client Meeting .............................................................................................2

3. Contract between Conrad Croson and Maria Gonzalez .............................................................3

4. Formatting Memorandum for Drafting Complaint ....................................................................4

5. Sample Complaint......................................................................................................................5

LIBRARY

1. Pennsylvania Rule of Civil Procedure 1019 ..............................................................................6

2. Pennsylvania Rule of Civil Procedure 1021 ..............................................................................6

3. Small County Rule of Civil Procedure 1301 .............................................................................7

4. Pennsylvania Unfair Trade Practices and Consumer Protection Law, 73 P.S. § 201-1 – 201-

9.3...............................................................................................................................................7

5. Nading v. Boice, 61 Pa. D&C 4th 353 (2003) ...........................................................................8

6. Johnson v. Hyundai Motor Am., 698 A.2d 631 (Pa. Super. 1997) ..........................................10

51
FILE

52
Memorandum

TO: Applicants
FROM: Luke Benjamin, Managing Partner
RE: Assignment to Draft Complaint
DATE: February 27, 2018

We represent Maria Gonzalez, a Pennsylvania resident, with an address of 10 Downing Street, Notown,
Small County, Pennsylvania, with regard to her claims against her contractor, Conrad Croson, and his
wife, Maggie Croson. Both Conrad and Maggie Croson are Pennsylvania residents who reside at 24
Sussex Drive, Notown, Small County, Pennsylvania. Conrad, a sole proprietor, also operates his only
place of business, i.e. custom cabinet making, out of his residence, 24 Sussex Drive, Notown, Small
County, Pennsylvania. As detailed in my notes from my meeting with Mrs. Gonzalez, which are
attached hereto, Mr. Croson negligently caused personal injuries to Mrs. Gonzalez, Mrs. Croson
negligently entrusted her vehicle to Mr. Croson allowing him to injure Mrs. Gonzalez, and Mr. Croson
also undertook actions that violate Pennsylvania law when he installed cabinets in Mrs. Gonzalez’s
kitchen.

Please use the facts contained in my typed notes from my meeting with Mrs. Gonzalez, along with the
materials in the attached Library to draft a complaint for negligence, negligent entrustment, and
violations of the Unfair Trade Practices and Consumer Protection Law (UTPCPL). The complaint will
be filed in the Pennsylvania Court of Common Pleas in Small County, Pennsylvania. I am still
reviewing the potential breach-of-contract claim and will insert an appropriate cause of action into your
draft complaint when I have finished my review. Thus, please do not address that issue in the
complaint. If I do, in fact, include such a claim, I will attach the contract to the complaint in accordance
with the rules of civil procedure. Thus, you should not do so.

Included in the attached File, in addition to my typed notes from my meeting with Mrs. Gonzalez, is the
contract between Mr. Croson and Mrs. Gonzalez, a formatting memorandum regarding drafting
complaints, and a sample complaint for your reference. The attached Library includes relevant caselaw,
portions of the Pennsylvania and Small County rules of civil procedure, and portions of the UTPCPL.
Do not rely on your personal knowledge of these issues, or on cases, rules, and statutes not included in
the Library. Instead you should draft your complaint only based upon the documents provided in the
File and the Library.

53
Memorandum

TO: Luke Benjamin, Managing Partner


FROM: Megan McDonough, Administrative Assistant
RE: Your Notes from Client Meeting
DATE: February 27, 2018

Mrs. Gonzalez, a retiree, resides at 10 Downing Street, Notown, Small County, Pennsylvania. She owns an older
home and wished to upgrade her kitchen. Because of the age and construction of her home, prefabricated cabinets
would not fit in her kitchen. In order to do the job the right way, Mrs. Gonzalez needed the expertise of someone
who makes custom cabinets. She was referred by an acquaintance to a contractor named Conrad Croson. On
December 1, 2017, Mr. Croson told Mrs. Gonzalez that he builds the finest and sturdiest custom cabinets in the
United States of America. That day, Mrs. Gonzalez signed a contract hiring Mr. Croson to replace the kitchen
cabinets in her home. He promised her that he would handcraft custom hardwood cabinets for her that would
look great and last a lifetime, and that he would make each cabinet on-site, by hand, to ensure a perfect fit. Mrs.
Gonzalez paid Mr. Croson in full, in advance, so that he could purchase the wood to make the cabinets. The
custom cabinets Mr. Croson agreed to build and install cost five times more than prefabricated cabinets.

On January 2, 2018, Mr. Croson came to build and install the custom cabinets. Per their arrangement, Mrs.
Gonzalez left a key under her doormat that morning so that Mr. Croson could gain entry to the kitchen while she
volunteered at her church. On her way home that day, Mrs. Gonzalez was surprised to pass a truck leaving her
driveway emblazoned with a “Kabinets-R-Us” logo. Kabinets-R-Us is a company known for making one-size-
fits-all prefabricated cabinets from poor quality materials. When Mrs. Gonzalez walked into her kitchen, she saw
several cardboard cartons with pictures of cabinets on the sides. There appeared to be one box for each cabinet
that Mr. Croson installed in her kitchen. Further, there was no extra wood lying around; no sawdust anywhere;
and no saw for cutting wood. Worse yet, the cabinets stick out farther than they should; there are gaps between
the cabinets and the walls; and the cabinets were made of particle board instead of hardwood. Mrs. Gonzalez
believes Mr. Conrad installed prefabricated cabinets instead of the custom made hardwood cabinets for which she
paid.

At approximately 2:30 p.m., Mrs. Gonzalez confronted Mr. Croson about the cabinets; he immediately grabbed
his tool bag and walked out the door, angrily saying, “These cabinets are great.” Mrs. Gonzalez followed Mr.
Croson to the blue Ford Explorer, owned by Mrs. Croson, trying to talk to him. She went to the open window on
the passenger side as Mr. Croson got into the driver side of the car. Mr. Croson started the car, looked directly at
Mrs. Gonzalez standing right next to the car and sped off. Upon doing so, he ran over Mrs. Gonzalez’s foot,
breaking it in several places. Mrs. Gonzalez was rushed to the hospital, where she underwent emergency surgery.
She is expected to have four more operations before her foot will be well enough for her to walk again, but, even
then, her activities will be limited for the rest of her life.

I have estimated the value of her claim for injuries to her foot to be $300,000, and she paid $55,000 for the
kitchen cabinets that she will have to tear out and replace.

I have also been able to ascertain that Mrs. Croson owns the blue Ford Explorer Mr. Croson was driving when he
ran over Mrs. Gonzalez’s foot; and this is the sixth time during the past two months that Mr. Croson has caused
injuries to someone in a motor-vehicle-related incident. I know the Croson’s next door neighbor; the neighbor
told me that the Crosons frequently fight about Mr. Croson’s poor driving, and how his poor driving has caused
many injuries in the past. On the morning of January 2, 2018, the neighbor overheard Mrs. Croson say that
“every time you use my car you injure someone.”

54
Contract for Custom Cabinets by Conrad Croson

This Agreement is between Conrad Croson (“Contractor”) and Maria Gonzalez (“Customer”).

Contractor agrees to build and install for Customer the finest and sturdiest custom cabinets in the
United States of America. Said cabinets will be handcrafted by Contractor at Customer’s home and
shall be made from hardwood. Said cabinets will look great and last a lifetime. By handcrafting custom
cabinets, made onsite, out of the finest and sturdiest materials, Contractor ensures that the cabinets will
be a perfect fit.

In exchange for the labor and materials provided by Contractor in conjunction with the
handcrafting and installation of the custom cabinets, Customer agrees to pay to Contractor the sum of
$55,000.00. Funds are due immediately so that Contractor can purchase the hardwood and other fine
materials required for building the custom cabinets in advance of the handcrafting and installation.

By signing below, Contractor acknowledges and agrees to both his obligations set forth above,
and receipt of full payment for the custom cabinets. By signing below, Customer agrees to pay
Contractor the sum set forth above, in advance of the cabinets being handcrafted and installed.

Contractor: Conrad Croson Customer: Maria Gonzalez

Date: 12/1/2017 Date: 12/1/2017

55
Big Law Firm

TO: All Associates and Law Clerks


FROM: Luke Benjamin, Managing Partner
RE: Formatting Memorandum for Drafting Complaint
DATE: February 1, 2017

Use the following guidelines for drafting a complaint:

1. A complaint is a pleading that sets forth a legal cause of action against one or more parties.

2. A paralegal will place a caption, containing all relevant information, at the top of your
complaint. The paralegal will also place an appropriate notice to plead, verification, and
signature block at the end of the complaint. Thus, do not include any of these items in the
complaint.

3. The complaint shall be divided into paragraphs numbered consecutively.

4. Each paragraph in the complaint shall contain as far as practicable only one material
allegation.

5. A material allegation is an assertion of fact that is essential to the claim.

6. Pennsylvania is a fact pleading jurisdiction in which a plaintiff must set forth concisely the
facts upon which a cause of action is based.

7. A complaint may contain more than one cause of action, and it may bring related causes of
action against more than one defendant. Each cause of action must be stated in a separate
count, and must include its own demand for relief. Thus, the complaint should have a
separate count (e.g., “Count I – Negligence”) for each cause of action.

8. Any claim(s) for relief must be in accordance with the rules of civil procedure.

A sample complaint (also with no caption, notice to plead, verification, venue, or signature
block) is attached hereto for your reference.

56
SAMPLE COMPLAINT

NOW COMES, the Plaintiff, Aaron Plastics, Inc., and for its complaint avers as follows:

1. Aaron Plastics, Inc. (hereinafter “Aaron”) is a Pennsylvania corporation with its principal
place of business at 1 Flickinger Lane, Big City, Kathryn County, Pennsylvania.
2. RES Electric Services, Inc. (hereinafter “RES”) is a Pennsylvania corporation with its
principal place of business at 1 Main Street, Big City, Kathryn County, Pennsylvania.
3. Aaron and RES are parties to that certain Electric Services Agreement dated September
2, 2010, with a term of ten years, and wherein RES agreed to supply electric services to
Aaron at a discounted rate, and Aaron agreed to pay for those discounted electric
services.
4. The Agreement, attached hereto as Exhibit A, required Aaron to pay to RES any amounts
due within twenty days of the billing date.
5. As a result of a change in accounting department personnel, Aaron neglected to make
timely payment of its July 2013 payment, which was due by August 21, 2013, and totaled
two-thousand five hundred dollars ($2,500), an average size bill.
6. On September 23, 2013, an employee of RES who was authorized to waive payment due
dates by virtue of being the Accounting Manager of RES, sent to Aaron an e-mail
communication that waived the time for payment, as well as remedies under the
Agreement.
7. The e-mail correspondence stated:

Our records indicate that we never received payment for electricity for the
month of July 2013. If you have a cancelled check for this payment,
please send me a copy or send payment as soon as possible to avoid late
charges.

8. On September 25, 2013, RES sent to Aaron a notice that it was discontinuing services
being provided under the Agreement; and RES stopped providing electricity to Aaron.
9. RES has refused to resume the provision of electric services to Aaron.
10. RES is barred from claiming Aaron’s failure to pay is a breach of the Agreement as a
result of the express waiver contained in the aforementioned September 23, 2013, e-mail
correspondence from RES’ Accounting Manager to Aaron.

BREACH OF CONTRACT – AARON v. RES


11. Each paragraph of this complaint is incorporated herein as if set forth at length.
12. The aforementioned conduct of RES constitutes a breach of the Agreement.
13. As a result of RES’ breach of the Agreement, RES is liable to Aaron for the damages
provided in the Agreement.

WHEREFORE, Aaron demands judgment in its favor and against RES in an amount in excess of
Fifty Thousand Dollars ($50,000).

57
LIBRARY

58
Pa. R.C.P. 1019. Contents of Pleadings. General and Specific Averments.

(a) The material facts on which a cause of action or defense is based shall be stated in a concise and
summary form.

(b) Averments of fraud or mistake shall be averred with particularity. Malice, intent, knowledge, and
other conditions of mind may be averred generally.

(c) In pleading the performance or occurrence of conditions precedent, it is sufficient to aver generally
that all conditions precedent have been performed or have occurred. A denial of such performance or
occurrence shall be made specifically and with particularity.

(d) In pleading an official document or official act, it is sufficient to identify it by reference and aver that
the document was issued or the act done in compliance with law.

(e) In pleading a judgment, order or decision of a domestic or foreign court, judicial or administrative
tribunal, or board, commission or officer, it is sufficient to aver the judgment, order or decision
without setting forth matter showing jurisdiction to render it.

(f) Averments of time, place and items of special damage shall be specifically stated.

(g) Any part of a pleading may be incorporated by reference in another part of the same pleading or in
another pleading in the same action. A party may incorporate by reference any matter of record in any
State or Federal court of record whose records are within the county in which the action is pending, or
any matter which is recorded or transcribed verbatim in the office of the prothonotary, clerk of any
court of record, recorder of deeds or register of wills of such county.

(h) When any claim or defense is based upon an agreement, the pleading shall state specifically if the
agreement is oral or written.

Official Note:

If the agreement is in writing, it must be attached to the pleading. See subdivision (i) of this rule.

(i) When any claim or defense is based upon a writing, the pleader shall attach a copy of the writing, or
the material part thereof, but if the writing or copy is not accessible to the pleader, it is sufficient so to
state, together with the reason, and to set forth the substance of the writing.

Pa. R.C.P. 1021. Claim for Relief. Determination of Amount in Controversy

(a) Any pleading demanding relief shall specify the relief sought . . .
***

(b) In counties having rules governing compulsory arbitration the plaintiff shall state whether the amount
claimed does or does not exceed the jurisdictional amount requiring arbitration referral by local rule.
***

59
Small County Local Rule of Civil Procedure 1301. Arbitration

All civil actions brought in the Court of Common Pleas of Small County in which the amount in controversy is
$50,000 or less shall first be submitted to arbitration and heard by a panel of three arbitrators selected from
members of the bar of this court in accordance with the provisions of this rule.

73 P.S. § 201-1 – 201-9.3

PENNSYLVANIA UNFAIR TRADE PRACTICES AND CONSUMER PROTECTION LAW

§ 201-1. Short title

This act shall be known and may be cited as the “Unfair Trade Practices and Consumer Protection Law.”

§ 201-2. Definitions

As used in this act.

***

(4) “Unfair methods of competition” and “unfair or deceptive acts or practices” mean any one or
more of the following:
(i) Passing off goods or services as those of another;
***

(v) Representing that goods or services have sponsorship, approval, characteristics, ingredients,
uses, benefits or quantities that they do not have or that a person has a sponsorship, approval,
status, affiliation or connection that he does not have;
***

(vii) Representing that goods or services are of a particular standard, quality or grade, or that
goods are of a particular style or model, if they are of another[.]

***

§ 201-3. Unlawful acts or practices; exclusions

Unfair methods of competition and unfair or deceptive acts or practices in the conduct of any trade or commerce
as defined by subclauses (i) through (xxi) of clause (4) of section [201-2] of this act . . . are hereby declared
unlawful.

§ 201-9.2. Private actions

(a) Any person who purchases or leases goods or services primarily for personal, family or household purposes
and thereby suffers any ascertainable loss of money or property, real or personal, as a result of the use or
employment by any person of a method, act or practice declared unlawful by section [201-3] of this act, may
bring a private action to recover actual damages or one hundred dollars ($100), whichever is greater. The court
may, in its discretion, award up to three times the actual damages sustained, but not less than one hundred dollars
($100), and may provide such additional relief as it deems necessary or proper. The court may award to the
plaintiff, in addition to other relief provided in this section, costs and reasonable attorney fees.

60
61 Pa. D. & C.4th 353 (Pa.Ct.Com.Pl. 2003)

Nading

v.

Boice

***
Before this court for consideration are the defendants[’] . . . preliminary objections filed in
response to the plaintiffs[’] complaint in the above captioned matter. For the following reasons,
defendants' preliminary objections are sustained in part and overruled in part.

***

Plaintiffs filed their complaint . . . alleging that they were injured as a result of a motor vehicle
accident with a van driven by defendant Kenneth Boice. Plaintiffs allege that . . . plaintiff David Nading
was driving his pickup truck east on State Route 68 near the intersection of Routes 68 and 268. Plaintiff
Zachary Nading was a passenger in the truck. Plaintiffs allege that the defendants' van was facing a
flashing red light and stop sign at the intersection of Routes 68 and 268. Defendants allegedly turned
into plaintiffs' lane of traffic in violation of the traffic signals. Defendants' van struck the passenger side
of plaintiffs' truck. Plaintiffs allege that defendant Elizabeth Boice owns the van driven by defendant
Kenneth Boice. Plaintiffs' complaint alleges two causes of action against defendants, negligence and
negligence per se. Defendants filed preliminary objections . . . arguing that the complaint fails to state a
cause of action as to Elizabeth Boice . . . . Plaintiffs filed a brief in response to defendants’ preliminary
objections, arguing that the owner of the vehicle bears liability for a defective or substandard vehicle.
Plaintiffs allege that the braking mechanism on the van was not in proper working order.

***

1. Demurrer -- Failure To State a Cause of Action Against Defendant Elizabeth Boice

Defendants argue that the complaint fails to state a cause of action against defendant Elizabeth
Boice. They argue that plaintiffs have not set forth any allegations that Ms. Boice was negligent.
Defendants also argue that plaintiffs' complaint fails to state a cause of action against Elizabeth Boice
for negligent entrustment. Plaintiffs argue that owners have liability for a defective or substandard
vehicle. Paragraph 14(d) of plaintiffs' complaint alleges that defendants were careless or negligent "in
failing to have the brakes and braking mechanism on said vehicle in proper working order and/or in
failing to properly and promptly operate the brakes and braking mechanism." Said paragraph is the sole
allegation in the complaint that relates to defendant Elizabeth Boice.

In order to state a valid cause of action for negligence, a plaintiff must plead four elements: "(1)
a duty or obligation recognized by the law that requires an actor to conform his actions to a standard of
conduct for the protection of others against unreasonable risks; (2) failure on the part of the defendant to
conform to that standard of conduct, i.e., a breach of duty; (3) a reasonably close causal connection
between the breach of duty and the injury sustained; and (4) actual loss or damages that result from the

61
breach." [citations omitted] "[A] mere spousal relationship is insufficient to impose liability upon a
spouse, as owner of a vehicle, due to the other spouse's negligent driving of that vehicle." [citation
omitted] The Pennsylvania Supreme Court stated the following with respect to an owner's duty to
maintain a vehicle:

‘Generally speaking, it is the duty of one operating a motor vehicle on the public
highways to see that it is in reasonably good condition and properly equipped, so that it
may be at all times controlled, and not become a source of danger to its occupants or to
other travelers. To this end, the owner or operator of a motor vehicle must exercise
reasonable care in the inspection of the machine and is chargeable with notice of
everything that such inspection would disclose.’

[citation omitted]

[Here], defendant Elizabeth Boice's duty to maintain the vehicle in working condition can be
inferred from the allegations contained in paragraph 14(d). Said paragraph also alleges that defendant
Elizabeth Boice breached her duty by failing to maintain the brakes on her vehicle in working condition.
Plaintiffs' complaint also alleges the negligence elements of cause and damages. As such, plaintiffs'
complaint states a cause of action against defendant Elizabeth Boice for negligence.

In order to plead a valid claim for negligent entrustment, a plaintiff must plead that the defendant
(1) permitted a third person, (2) to use a thing under the control of the defendant, and (3) that the
defendant knew or should have known that the third person intended to or was likely to use the thing in
such a way that would harm another. [citation omitted] Plaintiffs' complaint does not make any
allegations that set forth a claim for negligent entrustment against Ms. Boice. Therefore, plaintiffs'
complaint fails to set forth a cause of action for negligent entrustment.

Plaintiffs' complaint states a cause of action for negligence. It does not state a cause of action for
negligent entrustment. Therefore, defendants' preliminary objection as to defendant Elizabeth Boice is
overruled as to negligence and sustained as to negligent entrustment.

***

62
698 A.2d 631 (Pa.Super. 1997)

JOHNSON and JOHNSON,

v.

HYUNDAI MOTOR AMERICA and MCCAFFERTY HYUNDAI SALES, INC.,

***

MONTEMURO, Judge.

Appellants, Hyundai Motor America (Hyundai) and McCafferty Hyundai Sales, Inc.
(McCafferty) appeal from the . . . denial of their post-trial motions for judgment notwithstanding the
verdict (JNOV) and/or a new trial. For the reasons set forth below, we affirm. The facts giving rise to the
underlying action are as follows. On June 1, 1988, Appellees, Steven and DeLee Johnson, bought a
1988 Hyundai GLS sedan from McCafferty, an authorized Hyundai dealer and repair facility. The car
was manufactured by Hyundai and was equipped with an anti-theft device installed by McCafferty. At
the time of the purchase, Appellees were given a handbook that contained various warranty and
consumer information . . . .

***

When Appellees purchased their vehicle, they obtained a warranty from Hyundai for twelve
months or 12,500 miles. In addition to the manufacturer's warranty, they also separately purchased a
warranty from McCafferty for five years or 50,000 miles.

Shortly after purchasing their new Hyundai, Appellees began to experience serious difficulty
with its performance. The ensuing difficulties are aptly summarized by the trial court as follows:

[Approximately one month after the purchase of the Hyundai,] on July 10, 1988,
the car would not start when the key was placed in the ignition. The car was towed to
McCafferty. The mileage on the car at that time was 2,497 miles. McCafferty advised
plaintiffs that it had fixed the car, replaced the anti-theft device, and issued a work order
indicating the work done on the car. The repair order indicated that a fuse had been
replaced.

Five days later, on July 15, 1988, [Appellees] experienced the no start condition
again. Once again, the car was towed to McCafferty, and McCafferty purportedly fixed
the car. This time, McCafferty did not provide a repair receipt, despite the fact that
[Appellees] specifically asked for one. Also at this time, the anti-theft device was
removed from the car.

Seven days later, on July 22, 1988, [Appellees] experienced the no start condition
for the third time. At this point, the car had been driven 2,582 miles. Again the car was
towed to McCafferty. McCafferty purportedly repaired the car and issued a repair order.
The repair order indicated a fuse had been replaced.

63
Finally, on August 13, 1988, [Appellees] experienced the no start condition for
the fourth time. Now, the vehicle had 2,896 miles on it. The service order again indicated
that fuses had been replaced. Moreover, the service records indicate that the time spent
attempting to repair the car on the third and fourth attempts was about two tenths of an
hour, with minimal cost to the dealership. [citation omitted]

After the fourth no start condition, Appellees decided that they were not going to
accept the return of the vehicle. Instead, Appellees mailed letters to Consumer Affairs
and telephoned the office of Hyundai's president, but to no avail. As a result, Appellees
sought legal advice and brought suit against Hyundai and McCafferty under the
Pennsylvania Automobile Lemon Law ("Lemon Law") [citation omitted]; the
Pennsylvania Unfair Trade Practices and Consumer Protection Law ("UTPCPL")
[citation omitted] the Pennsylvania Uniform Commercial Code, Article II Sales [citation
omitted]; and the Magnuson-Moss Warranty Act [citation omitted].

***

For the eight years between August 15, 1988, when Appellees refused return of the vehicle, and
the January 25, 1996 trial date, the car remained untouched at McCafferty while Appellees dutifully paid
all of their monthly payments on the car loan. However, on January 24, 1996, one day before trial, two
master technicians from McCafferty, Mr. Richard DeFeo and Mr. Paul, inspected the car in a serious
effort to discover the problem. After six hours, they found the real difficulty. At trial, Mr. DeFeo
testified that as of the time of inspection, "it was pretty obvious that there was something not right
there." [citation omitted] His thorough inspection revealed that "a short to ground had caused the system
to burn" with respect to all of the no start problems. [citation omitted]

During trial, the court entered nonsuit on Appellees' claim under the Lemon Law because their
vehicle, although purchased in Pennsylvania, was registered in New Jersey and, therefore, did not meet
the prerequisites for eligibility under the Lemon Law. At the close of the trial, the jury answered special
interrogatories, finding both Appellants in breach of certain warranties, and finding that McCafferty had
made repairs or replacements on the vehicle of a nature or quality below the standard of that agreed to in
writing. The jury concluded, however, that neither Appellant acted fraudulently.

Following the damages phase of trial, the jury awarded $17,709.70 in compensatory damages.
The court then trebled this award, pursuant to its discretion under UTPCPL, for a total of $53,129.10.
The court also added to the award attorney's fees and costs of $11,500, and interest in the amount of
$2,878.40, resulting in a total verdict of $67,507.50.

***

Both Appellants present several issues for our review. Hyundai challenges the trial court's
imposition of joint and several liability as to both compensatory and treble damages, and the trial court's
award of treble damages where liability was based solely upon breach of warranty. McCafferty
challenges the jury's finding of liability under the UTPCPL, as well as the trial court's damages
calculation, award of attorney's fees, and its refusal to consider a cross-claim for indemnification. Both

64
Appellants challenge the trial court's award of treble damages absent a finding of fraud . . . .

***

After a thorough review of the record, the briefs of the parties, as well as applicable caselaw and
statutory authority, we conclude that the trial court has sufficiently addressed and properly disposed of
Appellants' issues regarding UTPCPL liability, attorney's fees, damages calculations, joint and several
liability as to compensatory damages, and indemnification. With respect to those issues, we agree with
the trial court's conclusion that neither JNOV nor a new trial is warranted, and, therefore, affirm on the
basis of the well-reasoned trial court Opinion. [footnote omitted] However, Appellants' claims
regarding the award of treble damages . . . warrant further discussion.

***

[W]e address Appellants' claims regarding treble damages. Under the UTPCPL,

[a]ny person who purchases . . . goods . . . primarily for personal, family or household
purposes and thereby suffers any ascertainable loss of money or property, real or
personal, as a result of the use or employment by any person of a method, act or practice
declared unlawful by section 3 of this act, may bring a private action to recover actual
damages or one hundred dollars ($100), whichever is greater. The court may, in its
discretion, award up to three times the actual damages sustained . . . and may provide
such additional relief as it deems necessary or proper.

73 P.S. § 201-9.2(a) (footnotes omitted). Appellants argue that as in the instant case, where liability
under UTPCPL is based solely upon a breach of warranty, [footnote omitted] a court may not award
treble damages absent a finding of egregious, fraudulent conduct. [citation omitted] Although the trial
court agreed with this assessment of the law, it found that Appellants' conduct was indeed "reckless,"
and, therefore, exercised its discretion under the UTPCPL and awarded treble damages. For the reasons
set forth below, we find no abuse of that discretion.

The purpose of the UTPCPL is to protect the public from fraud and unfair or deceptive business
practices. [citation omitted] Therefore, it is not surprising that most of the subsections enumerated
under the Act's heading of "unfair or deceptive acts or practices" involve situations such as: passing off
goods or services as those of another; disparaging the goods, services, or business of another by false or
misleading representation of fact; advertising goods or services with intent not to sell as advertised;
representing that goods are original or new if they are deteriorated, altered, or second-hand; or engaging
in "any other fraudulent conduct." [citation omitted]

Unlike most of the prohibited acts under the UTPCPL which "sound in tort/trespass," however,
Appellants contend that the provision which they were found guilty of violating "sounds strictly in
contract/assumpsit in the nature of a breach of warranty." [citation omitted] Therefore, since treble
damages are punitive in nature and, thus, not typically awarded in contract actions, Appellants argue that
the court must first find "outrageous" or "egregious" conduct before an award of treble damages is
appropriate for a breach of contract or warranty under the UTPCPL. [citation omitted] Appellants
conclude that since the jury specifically found that neither defendant engaged in any fraudulent conduct,

65
and there was no evidence of any outrageous or egregious conduct presented at trial, the trial court's
award of treble damages was erroneous. (Id.)

By advancing this argument, Appellants essentially urge this Court to adopt the reasoning of the
federal district courts which have interpreted the UTPCPL to require a finding of "outrageous conduct"
before awarding treble damages for breach of contract or warranty under the UTPCPL.

***

At the outset, we remind Appellants in the instant case that this Court is not bound by any federal
interpretation of Pennsylvania statutes. Although we acknowledge that, unlike most of the UTPCPL
provisions, the provisions violated by Appellants were solely breach of contract/warranty, it is clear that
the UTPCPL does not differentiate between "tort-like" violations and "contract-like" violations; rather,
all prohibited "unfair or deceptive acts or practices" are listed together in section § 201-2(4)(i)-(xvii).

It is also clear from a plain reading of the statute that upon the commission of a prohibited act in
any of the enumerated subsections in § 201-2(4), the trial court "may, in its discretion, award up to three
times the actual damages sustained, but not less than one hundred dollars ($100), and may provide such
additional relief as it deems necessary or proper." 73 P.S. § 201-9.2(a). The statute does not explicitly
require a finding of "outrageous conduct" before a trial court may award treble damages for a breach of
warranty violation under the UTPCPL. Rather, without providing further guidance, the legislature has
vested the trial court with considerable discretion in the imposition of treble damages. Although
Appellants would prefer otherwise, statutory construction is within the power of the legislature, and it is,
therefore, beyond the jurisdiction of this Court to reformulate the UTPCPL by inserting language which
differentiates among prohibitions and imposes specific prerequisites for recovery. Therefore, since the
legislature has not provided for a specific finding of outrageous conduct before an award of treble
damages for breach of contract/warranty may be imposed, we must trust that the courts will be guided
by the well-established, general principles of law governing punitive damages when exercising
discretion under the UTPCPL.

It is undisputed that the imposition of exemplary or treble damages is essentially punitive in


nature. The law of Pennsylvania clearly provides, however, that punitive damages are not recoverable in
an action solely based upon breach of contract. [citations omitted] Therefore, under the law of this
Commonwealth, a court may award punitive damages only if an actor's conduct was malicious, wanton,
willful, oppressive, or exhibited a reckless indifference to the rights of others. [citation omitted] It is
precisely these well-grounded principles of law that we expect the trial courts will follow when, as in the
instant case, exercising discretion and awarding treble damages for breach of contract/warranty under
the UTPCPL.

Indeed, in the instant case, it is clear that the trial court properly applied the law as outlined
above and concluded that Appellants' actions were "recklessly indifferent," thus warranting treble
damages. The trial court stated that there was "more than enough material in the record to support this
finding." [citation omitted] We agree.

For example, neither defendant took the time to discover successfully the real problem

66
with the car until one day before trial. Prior to this time, the car sat in a storage lot for
nearly 6 1/2 years with each defendant maintaining that the car was fixed. All the while,
the plaintiffs made their monthly payments on a new car they could not use. In the actual
attempts to fix the car, little money or time was spent [by Appellants]. Once the problem
was discovered, both sides continued to blame the other side, trying to escape blame.
McCafferty actually entered a counterclaim against the plaintiffs for storage costs and
maintained the claim even after they had discovered that the car was indeed in need of
repair and unsuitable for use.

[citation omitted] "[O]bviously neither of these defendants believed their customer; and, therefore, they
never spent the requisite time and attention to find the defect until one day before trial, and six years
after the sale." [citation omitted]

After reviewing the record, we agree with the trial court's conclusion and, therefore, find that it
did not abuse its discretion by awarding treble damages for Appellants' breach of contract/warranty
under the UTPCPL.

Order affirmed.

***

67
Question No. PT: Examiner’s Analysis

The applicant is assigned to draft a complaint. The complaint should include causes of action against Conrad Croson
for negligence and violations of the Unfair Trade Practices and Consumer Protection Law, and against Maggie
Croson for negligent entrustment. A formatting memorandum and sample complaint were included in the file, and
applicable rules of civil procedure were included in the Library, in order to provide Applicants with the framework
for drafting the complaint. Also provided to Applicants are: portions of the Unfair Trade Practices and Consumer
Protection Law (“UTPCPL”); and two cases setting forth the relevant law on negligence and negligent entrustment,
and background on the UTPCPL.

Formatting the Complaint 2 Points

Following instructions in assignment memoranda and the rules of civil procedure is an important skill every lawyer
must master. In this case, the formatting memorandum and rules of civil procedure provide the structure for drafting
a complaint. The applicant is expected to follow the memorandum and rules in order to draft the complaint. These
rules dictate the formatting and content of the complaint.

Several rules that a practitioner generally must follow have been intentionally omitted so that applicants are not
forced to focus mainly on structure/formatting instead of on analysis and applying of the law to the facts. Thus
applicants have been instructed not to include a caption; signature; verification; etc. However, applicants are asked
to draft their complaint using paragraphing; with a separate count for each cause of action; and they are asked to
include a claim for relief for each cause of action, taking into account the local rule governing arbitration.

Applicants are instructed to bring three causes of action: Negligence, violation of the Unfair Trade Practices and
Consumer Protection Law, and Negligent Entrustment. Each cause of action should be stated in a separate count
containing a demand for relief. Thus, the complaint should contain three separate counts with three separate
demands for relief.

Applicants should divide the complaint into consecutively numbered paragraphs, with each paragraph containing
as far as practicable only one material allegation.

Applicants should include a demand for relief for money damages for each cause of action and, for the UTPCPL
claim, should request treble damages, costs and reasonable attorney fees. The demand should also state whether
the amount claimed exceeds the jurisdictional amount requiring arbitration; in this case, fifty thousand dollars
($50,000.00). See Small County Rule of Civil Procedure 1301.

In support of each cause of action contained in the complaint, Applicants must include appropriate facts from the
materials provided. Credit will be given for facts provided in a fact section of the complaint, if Applicants choose
to include one, or if the facts are set forth within each cause of action.

Plaintiff’s Cause of Action for Negligence 6 Points

In order to state a valid cause of action for negligence, a plaintiff must plead four elements: "(1) a duty or obligation
recognized by the law that requires an actor to conform his actions to a standard of conduct for the protection of
others against unreasonable risks; (2) failure on the part of the defendant to conform to that standard of conduct,
i.e., a breach of duty; (3) a reasonably close causal connection between the breach of duty and the injury sustained;
and (4) actual loss or damages that result from the breach." Nading v. Boice

Applicants should include in the complaint averments alleging each element of negligence. For instance:

68
(1) Mr. Croson had a duty to refrain from running over Mrs. Gonzalez’s foot with the blue Ford Explorer
owned by Mrs. Croson;

(2) Mr. Croson breached his duty by running over Mrs. Gonzalez’s foot;

(3) Mr. Croson’s breach of his duty to refrain from running over Mrs. Gonzalez’s foot with the blue Ford
Explorer was the cause of Mrs. Gonzalez’s injuries and subsequent medical treatment; and

(4) Mrs. Gonzalez was forced to have emergency surgery on her foot, and will require four more
surgeries before she can walk again. Even then, her activities will be limited for the remainder of her
lifetime.

Applicants should set forth the details related to the time and place of the incident from which the cause of action
arose. For instance:

(1) On January 2, 2018, Mrs. Gonzalez arrived home and confronted Mr. Croson concerning the poor
quality cabinets he installed in her home.

(2) Mr. Croson left Mrs. Gonzalez’s house and got into the blue Ford Explorer owned by his wife, and
Mrs. Gonzalez followed him.

(3) Seeing that Mrs. Gonzalez was standing next to the passenger side of the car trying to talk to him, Mr.
Croson sped away, driving over Mrs. Gonzalez’s foot.

(4) Mrs. Gonzalez’s foot was broken in several places, and she was rushed to the hospital for emergency
surgery.

Applicants should include a demand for relief for negligence.

Plaintiff’s Cause of Action for Negligent Entrustment 4 Points

In order to plead a valid claim for negligent entrustment, “a plaintiff must plead that the defendant (1) permitted a
third person, (2) to use a thing under the control of the defendant, and (3) that the defendant knew or should have
known that the third person intended to or was likely to use the thing in such a way that would harm another.”
Nading v. Boice

Applicants should include in the complaint averments alleging each element of negligent entrustment. For instance:

(1) Mrs. Croson allowed Mr. Croson to operate the motor vehicle that she owns;

(2) Mrs. Croson, as the owner of the vehicle, had control over the vehicle when she allowed Mr. Croson
to use it; and

(3) Because Mr. Croson caused injuries to people in five other motor vehicle related incidents in the past
two months, Mrs. Croson knew or should have known that Mr. Croson was likely to harm another
person with the vehicle. In fact, Mrs. Croson was overheard by a neighbor on the day of the incident
stating “every time you use my car you injure someone.”

Despite the knowledge that Mr. Croson was likely to injure someone, Mrs. Croson nonetheless allowed Mr. Croson
to operate the blue Ford Explorer that she owned, thereby negligently entrusting the vehicle to Mr. Croson, resulting
in the severe injuries to Mrs. Gonzalez.

Applicants should include a demand for relief for negligent entrustment.

Plaintiff’s Cause of Action under the UTPCPL 8 Points

69
Applicants should include in the complaint averments supporting a claim under the UTPCPL. For instance:

Section 201-3 of the UTPCPL makes it unlawful to engage in any of the following actions, described in Section
201-2:

(j) Passing off goods or services as those of another;

(v) Representing that goods or services have sponsorship, approval, characteristics, ingredients,
uses, benefits or quantities that they do not have or that a person has a sponsorship, approval,
status, affiliation or connection that he does not have; and

(vii) Representing that goods or services are of a particular standard, quality or grade, or that
goods are of a particular style or model, if they are of another.

On December 1, 2017, Mr. Croson made numerous representations, both orally and in a contract, to Mrs. Gonzalez
concerning the origin, quality, characteristics, benefits, and standards of the kitchen cabinets he intended to install
in her kitchen.

Mr. Croson represented to Mrs. Gonzalez that his custom hardwood cabinets were the finest and sturdiest custom
cabinets in the United States of America; that they would be handcrafted custom hardwood cabinets; that the
cabinets would look great and last a lifetime; and that they would fit perfectly in her kitchen.

Mr. Croson represented to Mrs. Gonzalez that the cabinets he would install in her kitchen would be of the following
standard, quality and grade: they would be handmade; custom cabinets; hardwood cabinets; that would be the finest
and sturdiest custom cabinets in the entire United States of America.

In reality, on January 2, 2018, Mr. Croson provided store bought, cheap quality, ill-fitting, particle board cabinets
and installed them in her kitchen.

The cabinets Mr. Croson installed on January 2, 2018, were not handmade; were not custom cabinets; were not
hardwood cabinets; and were not the finest and sturdiest custom cabinets in the entire country.

Mrs. Gonzalez paid Mr. Croson $55,000, in advance, to install the custom hardwood cabinets he promised to
handcraft in her kitchen; she required custom cabinets because prefabricated cabinets will not fit in her older home.

Mr. Croson represented to Mrs. Gonzalez that he was making and installing in her kitchen custom hardwood
cabinets; instead, he bought cabinets from Kabinets-R-Us and installed them in her kitchen, engaging in unlawful
conduct under the UTPCPL, Section 201-2(4)(i).

By making these misrepresentations concerning the characteristics, uses, and benefits of the cabinets, Mr. Croson
engaged in unlawful conduct under the UTPCPL, Section 201-2(4)(v).

By making the misrepresentations concerning the standard, quality and grade of cabinets, Mr. Croson engaged in
unlawful conduct under the UTPCPL, Section 201-2(4)(vii).

Mrs. Gonzalez purchased and paid for the cabinets for use in her home kitchen, or for personal, family and
household purposes.

Mrs. Gonzalez suffered an ascertainable loss of money in connection with her purchase of the cabinets, as she paid
$55,000 for the custom hardwood cabinets that were never made or installed.

Mrs. Gonzalez is eligible under Section 201.9.2 of the UTPCPL to bring a private action against Mr. Croson, and
may be entitled to treble damages, costs, and reasonable attorney fees from Mr. Croson. Johnson v. Hyundai

Applicants should include a demand for relief for violations of the UTPCPL, including a demand for treble damages,
costs, and attorney’s fees. Johnson v. Hyundai

70

You might also like